专栏名称: 计量经济圈
记录一个我们生活在其中的时代社会,囊括的主题如下:经济、社会、世界和计量工具。
目录
相关文章推荐
首席商业评论  ·  辛巴对战小杨哥,胜者为王,败寇亦伤 ·  16 小时前  
大猫财经  ·  带走500亿,丁玉梅也过不好余生…… ·  3 天前  
经济观察报  ·  选AI苹果还是三折叠华为 ·  6 天前  
51好读  ›  专栏  ›  计量经济圈

新学期: TOP5刊AER, ECM, QJE 24年1-9月全部文章信息, 包括摘要, 必须快速浏览101遍

计量经济圈  · 公众号  · 财经  · 2024-09-02 00:00

正文

凡是搞计量经济的,都关注这个号了

邮件:[email protected]

所有计量经济圈方法论丛的code程序, 宏微观数据库和各种软件都放在社群里.欢迎到计量经济圈社群交流访问.

鉴于“1.我愿为在AER上发表一篇论文而牺牲一条右臂!”“2.发表TOP5的难度平均约为UTD24的5倍”,因此在开学季辛苦整理了TOP5刊QJE、ECM和AER2024年1月-9月全部文章信息,包括摘要。
接着“新学期: TOP5刊JPE 24年1-8月全部文章信息, 包括摘要, 必须快速浏览100遍”,
Quarterly journal of Economics 2024年1月-8月文章信息:
Monica P Bhatt, Sara B Heller, Max Kapustin, Marianne Bertrand, Christopher Blattman, Predicting and Preventing Gun Violence: An Experimental Evaluation of READI Chicago, The Quarterly Journal of Economics, Volume 139, Issue 1, February 2024, Pages 1–56, https://doi.org/10.1093/qje/qjad031
Gun violence is the most pressing public safety problem in U.S. cities. We report results from a randomized controlled trial (N = 2,456) of a community-researcher partnership called the Rapid Employment and Development Initiative (READI) Chicago. The program offered an 18-month job alongside cognitive behavioral therapy and other social support. Both algorithmic and human referral methods identified men with strikingly high scope for gun violence reduction: for every 100 people in the control group, there were 11 shooting and homicide victimizations during the 20-month outcome period. Fifty-five percent of the treatment group started programming, comparable to take-up rates in programs for people facing far lower mortality risk. After 20 months, there is no statistically significant change in an index combining three measures of serious violence, the study’s primary outcome. Yet there are signs that this program model has promise. One of the three measures, shooting and homicide arrests, declined 65% (p = .13 after multiple-testing adjustment). Because shootings are so costly, READI generated estimated social savings between $182,000 and $916,000 per participant (p = .03), implying a benefit-cost ratio between 4:1 and 18:1. Moreover, participants referred by outreach workers—a prespecified subgroup—saw enormous declines in arrests and victimizations for shootings and homicides (79% and 43%, respectively) which remain statistically significant even after multiple-testing adjustments. These declines are concentrated among outreach referrals with higher predicted risk, suggesting that human and algorithmic targeting may work better together.
Robert Collinson, John Eric Humphries, Nicholas Mader, Davin Reed, Daniel Tannenbaum, Winnie van Dijk, Eviction and Poverty in American Cities, The Quarterly Journal of Economics, Volume 139, Issue 1, February 2024, Pages 57–120, https://doi.org/10.1093/qje/qjad042
More than two million U.S. households have an eviction case filed against them each year. Policy makers at the federal, state, and local levels are increasingly pursuing policies to reduce the number of evictions, citing harm to tenants and high public expenditures related to homelessness. We study the consequences of eviction for tenants using newly linked administrative data from two major urban areas: Cook County (which includes Chicago) and New York City. We document that before housing court, tenants experience declines in earnings and employment and increases in financial distress and hospital visits. These pre trends pose a challenge for disentangling correlation and causation. To address this problem, we use an instrumental variables approach based on cases randomly assigned to judges of varying leniency. We find that an eviction order increases homelessness and hospital visits and reduces earnings, durable goods consumption, and access to credit in the first two years. Effects on housing and labor market outcomes are driven by effects for female and Black tenants. In the longer run, eviction increases indebtedness and reduces credit scores.
Adrien Auclert, Rodolfo Rigato, Matthew Rognlie, Ludwig Straub, New Pricing Models, Same Old Phillips Curves?, The Quarterly Journal of Economics, Volume 139, Issue 1, February 2024, Pages 121–186, https://doi.org/10.1093/qje/qjad041
We show that in a broad class of menu cost models, the first-order dynamics of aggregate inflation in response to arbitrary shocks to aggregate costs are nearly the same as in Calvo models with suitably chosen Calvo adjustment frequencies. We first prove that the canonical menu cost model is first-order equivalent to a mixture of two time-dependent models, which reflect the extensive and intensive margins of price adjustment. We then show numerically that in any plausible parameterization, this mixture is well approximated by a single Calvo model. This close numerical fit carries over to other standard specifications of menu cost models. Thus, for shocks that are not too large, the Phillips curve for a menu cost model looks like the New Keynesian Phillips curve, but with a higher slope.
Abe Dunn, Joshua D Gottlieb, Adam Hale Shapiro, Daniel J Sonnenstuhl, Pietro Tebaldi, A Denial a Day Keeps the Doctor Away, The Quarterly Journal of Economics, Volume 139, Issue 1, February 2024, Pages 187–233, https://doi.org/10.1093/qje/qjad035
Who bears the consequences of administrative problems in health care? We use data on repeated interactions between a large sample of U.S. physicians and many different insurers to document the complexity of health care billing, and estimate its economic costs for doctors and consequences for patients. Observing the back-and-forth sequences of claim denials and resubmissions for past visits, we can estimate physicians’ costs of haggling with insurers to collect payments. Combining these costs with the revenue never collected, we estimate that physicians lose 18% of Medicaid revenue to billing problems, compared with 4.7% for Medicare and 2.4% for commercial insurers. Identifying off of physician movers and practices that span state boundaries, we find that physicians respond to billing problems by refusing to accept Medicaid patients in states with more severe billing hurdles. These hurdles are quantitatively just as important as payment rates for explaining variation in physicians’ willingness to treat Medicaid patients. We conclude that administrative frictions have first-order costs for doctors, patients, and equality of access to health care. We quantify the potential economic gains—in terms of reduced public spending or increased access to physicians—if these frictions could be reduced and find them to be sizable.
Mathilde Muñoz, Trading Nontradables: The Implications of Europe’s Job-Posting Policy, The Quarterly Journal of Economics, Volume 139, Issue 1, February 2024, Pages 235–304, https://doi.org/10.1093/qje/qjad032
This article examines the labor market implications of the EU posting policy, a large temporary migration program facilitated by the liberalization of the free provision of services in Europe. Posting allows EU firms to send (post) their employees abroad to export customer-facing services. Combining administrative data and quasi-experimental policy variation, I find that the policy permanently increased total factor mobility in Europe without crowding out traditional migration. This result suggests that unrealized gains from trade in factor services remained despite the absence of regulatory barriers to trade and migration in the EU. Furthermore, posted workers are mostly sent from low-wage countries to perform manual tasks in sectors formerly insulated from trade, and they represent a substantial share of EU migrant workers. In receiving countries, posting had persistent negative effects on employment for domestic workers in the more exposed sectors and local labor markets, but it had no effects on domestic wages. In low-wage sending countries, firms in formerly “nontradable” sectors experienced increased sales, profits, and tax payments when exporting services through posting. Posted workers earn more once sent abroad but remain paid at lower wages than comparable domestic workers in the receiving country. Wage gains for posted workers are mostly explained by minimum wages enforced by the EU policy, highlighting the role of labor market regulations in shaping the way gains from globalization are shared between labor and capital owners in origin countries.
Isaiah Andrews, Toru Kitagawa, Adam McCloskey, Inference on Winners, The Quarterly Journal of Economics, Volume 139, Issue 1, February 2024, Pages 305–358, https://doi.org/10.1093/qje/qjad043
Policy makers, firms, and researchers often choose among multiple options based on estimates. Sampling error in the estimates used to guide choice leads to a winner’s curse, since we are more likely to select a given option precisely when we overestimate its effectiveness. This winner’s curse biases our estimates for selected options upward and can invalidate conventional confidence intervals. This article develops estimators and confidence intervals that eliminate this winner’s curse. We illustrate our results by studying selection of job-training programs based on estimated earnings effects and selection of neighborhoods based on estimated economic opportunity. We find that our winner’s curse corrections can make an economically significant difference to conclusions but still allow informative inference.
Mohammad Akbarpour, Eric Budish, Piotr Dworczak, Scott Duke Kominers, An Economic Framework for Vaccine Prioritization, The Quarterly Journal of Economics, Volume 139, Issue 1, February 2024, Pages 359–417, https://doi.org/10.1093/qje/qjad022
We propose an economic framework for determining the optimal allocation of a scarce supply of vaccines that become gradually available during a public health crisis, such as the COVID-19 pandemic. Agents differ in observable and unobservable characteristics, and the designer maximizes a social welfare function over all feasible mechanisms—accounting for agents’ characteristics, as well as their endogenous behavior in the face of the pandemic. The framework emphasizes the role of externalities and incorporates equity as well as efficiency concerns. Our results provide an economic justification for providing vaccines immediately and for free to some groups of agents, while at the same time showing that a carefully constructed pricing mechanism can improve outcomes by screening for individuals with the highest private and social benefits of receiving the vaccine. The solution casts light on the classic question of whether prices or priorities should be used to allocate scarce public resources under externalities and equity concerns.
David Atkin, Benjamin Faber, Thibault Fally, Marco Gonzalez-Navarro, Measuring Welfare and Inequality with Incomplete Price Information, The Quarterly Journal of Economics, Volume 139, Issue 1, February 2024, Pages 419–475, https://doi.org/10.1093/qje/qjad037
We propose and implement a new approach that allows us to estimate income-specific changes in household welfare in contexts where well-measured prices are not available for important subsets of consumption. Using rich but widely available expenditure survey microdata, we show that we can recover income-specific equivalent and compensating variations from horizontal shifts in what we call “relative Engel curves”—as long as preferences fall within the broad quasi-separable class (Gorman 1970, 1976). Our approach is flexible enough to allow for nonparametric estimation at each point of the income distribution. We apply the methodology to estimate inflation and welfare changes in rural India between 1987 and 2000. Our estimates reveal that lower rates of inflation for the rich erased the real income convergence found in the existing literature that uses the subset of consumption with well-measured prices to calculate inflation.
Xavier Jaravel, Danial Lashkari, Measuring Growth in Consumer Welfare with Income-Dependent Preferences: Nonparametric Methods and Estimates for the United States, The Quarterly Journal of Economics, Volume 139, Issue 1, February 2024, Pages 477–532, https://doi.org/10.1093/qje/qjad039
How should we measure changes in consumer welfare given observed data on prices and expenditures? This article proposes a nonparametric approach that holds under arbitrary preferences that may depend on observable consumer characteristics, for example, when expenditure shares vary with income. Using total expenditures under a constant set of prices as our money metric for real consumption (welfare), we derive a principled measure of real consumption growth featuring a correction term relative to conventional measures. We show that the correction can be nonparametrically estimated with an algorithm leveraging the observed, cross-sectional relationship between household-level price indices and household characteristics such as income. We demonstrate the accuracy of our algorithm in simulations. Applying our approach to data from the United States, we find that the magnitude of the correction can be large because of the combination of fast growth and lower inflation for income-elastic products. Setting reference prices in 2019, we find that (i) the uncorrected measure underestimates average real consumption per household in 1955 by 11.5%, and (ii) the correction reduces the annual growth rate from 1955 to 2019 by 18 basis points, which is larger than the well-known “expenditure-switching bias” over the same time horizon.
David R Baqaee, Ariel T Burstein, Yasutaka Koike-Mori, Measuring Welfare by Matching Households across Time, The Quarterly Journal of Economics, Volume 139, Issue 1, February 2024, Pages 533–573, https://doi.org/10.1093/qje/qjad038
The money metric utility function is an essential tool for calculating welfare-relevant growth and inflation. We show how to recover it from repeated cross-sectional data without making parametric assumptions about preferences. We do this by solving the following recursive problem. Given compensated demand, we construct money metric utility by integration. Given money metric utility, we construct compensated demand by matching households over time whose money metric utility value is the same. We illustrate our method using household consumption survey data from the United Kingdom from 1974 to 2017 and find that real consumption calculated using official aggregate inflation statistics overstates money metric utility in 1974 pounds for the poorest households by around 0.5% a year and understates it by around a third of a percentage point per year for the richest households. We extend our method to allow for missing or mismeasured prices, assuming preferences are separable between goods with well-measured prices and the rest. We discuss how our results change if the prices of some service sectors are mismeasured.
Marcella Alsan, Maya Durvasula, Harsh Gupta, Joshua Schwartzstein, Heidi Williams, Representation and Extrapolation: Evidence from Clinical Trials, The Quarterly Journal of Economics, Volume 139, Issue 1, February 2024, Pages 575–635, https://doi.org/10.1093/qje/qjad036
This article examines the consequences and causes of low enrollment of Black patients in clinical trials. We develop a simple model of similarity-based extrapolation that predicts that evidence is more relevant for decision-making by physicians and patients when it is more representative of the group being treated. This generates the key result that the perceived benefit of a medicine for a group depends not only on the average benefit from a trial but also on the share of patients from that group who were enrolled in the trial. In survey experiments, we find that physicians who care for Black patients are more willing to prescribe drugs tested in representative samples, an effect substantial enough to close observed gaps in the prescribing rates of new medicines. Black patients update more on drug efficacy when the sample that the drug is tested on is more representative, reducing Black-white patient gaps in beliefs about whether the drug will work as described. Despite these benefits of representative data, our framework and evidence suggest that those who have benefited more from past medical breakthroughs are less costly to enroll in the present, leading to persistence in who is represented in the evidence base.
Max Risch, Does Taxing Business Owners Affect Employees? Evidence From A Change in the Top Marginal Tax Rate, The Quarterly Journal of Economics, Volume 139, Issue 1, February 2024, Pages 637–692, https://doi.org/10.1093/qje/qjad040
Debates about the taxation of business owners often center on the distributional effects of these taxes, particularly the degree to which they affect workers. Drawing on a new linked owner-firm-worker data set created from U.S. administrative tax records, I analyze how an increase in the top marginal tax rate faced by business owners affected the earnings of their employees. I use panel difference-in-differences methods to compare the earnings of workers in similar firms but whose owners were differentially exposed to the tax increase. I estimate that 11–18 cents per dollar of new business income tax liability was passed through to employee earnings. I find no change in employment in response to the tax increase. The responses were generally associated with lower earnings growth, not changes in workforce composition. The burden was not borne equally by all workers. Essentially all of the workers’ share of the burden was borne by those in the top 30% of the earnings distribution, highlighting that the ultimate distributional effects of the policy depend not only on the share of the burden borne by workers but on the shares borne by different types of workers. Furthermore, since the owners bore the majority of the burden, the policy resulted in a decrease in after-tax earnings inequality between top-bracket owners and lower-bracket workers. I discuss the implications of the findings for the mediating labor market mechanisms and for welfare analyses of income taxation using a marginal value of public funds framework.
Ellora Derenoncourt, Chi Hyun Kim, Moritz Kuhn, Moritz Schularick, Wealth of Two Nations: The U.S. Racial Wealth Gap, 1860–2020, The Quarterly Journal of Economics, Volume 139, Issue 2, May 2024, Pages 693–750, https://doi.org/10.1093/qje/qjad044
The racial wealth gap is the largest of the economic disparities between Black and white Americans, with a white-to-Black per capita wealth ratio of 6 to 1. It is also among the most persistent. In this article, we construct the first continuous series on white-to-Black per capita wealth ratios from 1860 to 2020, drawing on historical census data, early state tax records, and historical waves of the Survey of Consumer Finances, among other sources. Incorporating these data into a parsimonious model of wealth accumulation for each racial group, we document the role played by initial conditions, income growth, savings behavior, and capital returns in the evolution of the gap. Given vastly different starting conditions under slavery, racial wealth convergence would remain a distant scenario, even if wealth-accumulating conditions had been equal across the two groups since Emancipation. Relative to this equal-conditions benchmark, we find that observed convergence has followed an even slower path over the past 150 years, with convergence stalling after 1950. Since the 1980s, the wealth gap has widened again as capital gains have predominantly benefited white households, and convergence via income growth and savings has come to a halt.
Jens Ludwig, Sendhil Mullainathan, Machine Learning as a Tool for Hypothesis Generation, The Quarterly Journal of Economics, Volume 139, Issue 2, May 2024, Pages 751–827, https://doi.org/10.1093/qje/qjad055
While hypothesis testing is a highly formalized activity, hypothesis generation remains largely informal. We propose a systematic procedure to generate novel hypotheses about human behavior, which uses the capacity of machine learning algorithms to notice patterns people might not. We illustrate the procedure with a concrete application: judge decisions about whom to jail. We begin with a striking fact: the defendant’s face alone matters greatly for the judge’s jailing decision. In fact, an algorithm given only the pixels in the defendant’s mug shot accounts for up to half of the predictable variation. We develop a procedure that allows human subjects to interact with this black-box algorithm to produce hypotheses about what in the face influences judge decisions. The procedure generates hypotheses that are both interpretable and novel: they are not explained by demographics (e.g., race) or existing psychology research, nor are they already known (even if tacitly) to people or experts. Though these results are specific, our procedure is general. It provides a way to produce novel, interpretable hypotheses from any high-dimensional data set (e.g., cell phones, satellites, online behavior, news headlines, corporate filings, and high-frequency time series). A central tenet of our article is that hypothesis generation is a valuable activity, and we hope this encourages future work in this largely “prescientific” stage of science.
Raj Chetty, John N Friedman, Michael Stepner, Opportunity Insights Team , The Economic Impacts of COVID-19: Evidence from a New Public Database Built Using Private Sector Data, The Quarterly Journal of Economics, Volume 139, Issue 2, May 2024, Pages 829–889, https://doi.org/10.1093/qje/qjad048
We build a publicly available database that tracks economic activity in the United States at a granular level in real time using anonymized data from private companies. We report weekly statistics on consumer spending, business revenues, job postings, and employment rates disaggregated by county, sector, and income group. Using the publicly available data, we show how the COVID-19 pandemic affected the economy by analyzing heterogeneity in its effects across subgroups. High-income individuals reduced spending sharply in March 2020, particularly in sectors that require in-person interaction. This reduction in spending greatly reduced the revenues of small businesses in affluent, dense areas. Those businesses laid off many of their employees, leading to widespread job losses, especially among low-wage workers in such areas. High-wage workers experienced a V-shaped recession that lasted a few weeks, whereas low-wage workers experienced much larger, more persistent job losses. Even though consumer spending and job postings had recovered fully by December 2021, employment rates in low-wage jobs remained depressed in areas that were initially hard hit, indicating that the temporary fall in labor demand led to a persistent reduction in labor supply. Building on this diagnostic analysis, we evaluate the effects of fiscal stimulus policies designed to stem the downward spiral in economic activity. Cash stimulus payments led to sharp increases in spending early in the pandemic, but much smaller responses later in the pandemic, especially for high-income households. Real-time estimates of marginal propensities to consume provided better forecasts of the impacts of subsequent rounds of stimulus payments than historical estimates. Overall, our findings suggest that fiscal policies can stem secondary declines in consumer spending and job losses, but cannot restore full employment when the initial shock to consumer spending arises from health concerns. More broadly, our analysis demonstrates how public statistics constructed from private sector data can support many research and real-time policy analyses, providing a new tool for empirical macroeconomics.
Jiafeng Chen, Jonathan Roth, Logs with Zeros? Some Problems and Solutions, The Quarterly Journal of Economics, Volume 139, Issue 2, May 2024, Pages 891–936, https://doi.org/10.1093/qje/qjad054
When studying an outcome Y that is weakly positive but can equal zero (e.g., earnings), researchers frequently estimate an average treatment effect (ATE) for a “log-like” transformation that behaves like log (Y) for large Y but is defined at zero (e.g., log (1 + Y), arcsinh(�)⁠). We argue that ATEs for log-like transformations should not be interpreted as approximating percentage effects, since unlike a percentage, they depend on the units of the outcome. In fact, we show that if the treatment affects the extensive margin, one can obtain a treatment effect of any magnitude simply by rescaling the units of Y before taking the log-like transformation. This arbitrary unit dependence arises because an individual-level percentage effect is not well-defined for individuals whose outcome changes from zero to nonzero when receiving treatment, and the units of the outcome implicitly determine how much weight the ATE for a log-like transformation places on the extensive margin. We further establish a trilemma: when the outcome can equal zero, there is no treatment effect parameter that is an average of individual-level treatment effects, unit invariant, and point identified. We discuss several alternative approaches that may be sensible in settings with an intensive and extensive margin, including (i) expressing the ATE in levels as a percentage (e.g., using Poisson regression), (ii) explicitly calibrating the value placed on the intensive and extensive margins, and (iii) estimating separate effects for the two margins (e.g., using Lee bounds). We illustrate these approaches in three empirical applications.
Abi Adams-Prassl, Kristiina Huttunen, Emily Nix, Ning Zhang, Violence against Women at Work, The Quarterly Journal of Economics, Volume 139, Issue 2, May 2024, Pages 937–991, https://doi.org/10.1093/qje/qjad045
We link every police report in Finland to administrative data to identify violence between colleagues and the economic consequences for victims, perpetrators, and firms. This new approach to observe when one colleague attacks another overcomes previous data constraints limiting evidence on this phenomenon to self-reported surveys that do not identify perpetrators. We document large, persistent labor market effects of between-colleague violence on victims and perpetrators. Male perpetrators experience substantially weaker consequences after attacking female colleagues. Perpetrators’ relative economic power in male-female violence partly explains this asymmetry. Turning to broader implications for firm recruitment and retention, we find that male-female violence causes a decline in the proportion of women at the firm, both because fewer new women are hired and current female employees leave. Management plays a key role in mediating the effects on the wider workforce. Only male-managed firms lose women. Female-managed firms exhibit a key difference relative to male-managed firms: male perpetrators are less likely to remain employed after attacking their female colleagues.
Maggie Shi, Monitoring for Waste: Evidence from Medicare Audits, The Quarterly Journal of Economics, Volume 139, Issue 2, May 2024, Pages 993–1049, https://doi.org/10.1093/qje/qjad049
This article examines the trade-offs of monitoring for wasteful public spending. By penalizing unnecessary spending, monitoring improves the quality of public expenditure and incentivizes firms to invest in compliance technology. I study a large Medicare program that monitored for unnecessary health care spending and consider its effect on government savings, provider behavior, and patient health. Every dollar Medicare spent on monitoring generated $24–$29 in government savings. The majority of savings stem from the deterrence of future care, rather than reclaimed payments from prior care. I do not find evidence that the health of the marginal patient is harmed, indicating that monitoring primarily deters low-value care. Monitoring does increase provider administrative costs, but these costs are mostly incurred up-front and include investments in technology to assess the medical necessity of care.
Christopher Campos, Caitlin Kearns, The Impact of Public School Choice: Evidence from Los Angeles’s Zones of Choice, The Quarterly Journal of Economics, Volume 139, Issue 2, May 2024, Pages 1051–1093, https://doi.org/10.1093/qje/qjad052
Does a school district that expands school choice provide better outcomes for students than a neighborhood-based assignment system? This article studies the Zones of Choice (ZOC) program, a school choice initiative of the Los Angeles Unified School District (LAUSD) that created small high school markets in some neighborhoods but left attendance zone boundaries in place throughout the rest of the district. We study market-level effects of choice on student achievement and college enrollment using a difference-in-differences design. Student outcomes in ZOC markets increased markedly, narrowing achievement and college enrollment gaps between ZOC neighborhoods and the rest of the district. The effects of ZOC are larger for schools exposed to more competition, supporting the notion that competition is a key channel. Demand estimates suggest families place substantial weight on schools’ academic quality, providing schools with competition-induced incentives to improve their effectiveness. The evidence demonstrates that public school choice programs have the potential to improve school quality and reduce neighborhood-based disparities in educational opportunity.
Jessica A Wachter, Michael Jacob Kahana, A Retrieved-Context Theory of Financial Decisions, The Quarterly Journal of Economics, Volume 139, Issue 2, May 2024, Pages 1095–1147, https://doi.org/10.1093/qje/qjad050
Studies of human memory indicate that features of an event evoke memories of prior associated contextual states, which in turn become associated with the current event’s features. This retrieved-context mechanism allows the remote past to influence the present, even as agents gradually update their beliefs about their environment. We apply a version of retrieved-context theory, drawn from the literature on human memory, to explain three types of evidence in the financial economics literature: the role of early life experience in shaping investment choices, occurrence of financial crises, and the effect of fear on asset allocation. These applications suggest a recasting of neoclassical rational expectations in terms of beliefs as governed by principles of human memory.
Ali Hortaçsu, Olivia R Natan, Hayden Parsley, Timothy Schwieg, Kevin R Williams, Organizational Structure and Pricing: Evidence from a Large U.S. Airline, The Quarterly Journal of Economics, Volume 139, Issue 2, May 2024, Pages 1149–1199, https://doi.org/10.1093/qje/qjad051
Firms facing complex objectives often decompose the problems they face, delegating different parts of the decision to distinct subunits. Using comprehensive data and internal models from a large U.S. airline, we establish that airline pricing is not well approximated by a model of the firm as a unitary decision maker. We show that observed prices, however, can be rationalized by accounting for organizational structure and for the decisions by departments that are tasked with supplying inputs to the observed pricing heuristic. Simulating the prices the firm would charge if it were a rational, unitary decision maker results in lower welfare than we estimate under observed practices. Finally, we discuss why counterfactual estimates of welfare and market power may be biased if prices are set through decomposition, but we instead assume that they are set by unitary decision makers.
Paul L E Grieco, Charles Murry, Ali Yurukoglu, The Evolution of Market Power in the U.S. Automobile Industry, The Quarterly Journal of Economics, Volume 139, Issue 2, May 2024, Pages 1201–1253, https://doi.org/10.1093/qje/qjad047
We construct measures of industry performance and welfare in the U.S. automobile market from 1980 to 2018. We estimate a demand model using product-level data on market shares, prices, and attributes, and consumer-level data on demographics, purchases, and stated second choices. We estimate marginal costs assuming Nash-Bertrand pricing. We relate trends in consumer welfare and markups to trends in market structure and the composition of products. Although real prices rose, we find that markups decreased substantially, and the fraction of total surplus accruing to consumers increased. Consumer welfare increased over time due to improved product quality and improved production technology.
Jérôme Adda, Marco Ottaviani, Grantmaking, Grading on a Curve, and the Paradox of Relative Evaluation in Nonmarkets, The Quarterly Journal of Economics, Volume 139, Issue 2, May 2024, Pages 1255–1319, https://doi.org/10.1093/qje/qjad046
The article develops a model of nonmarket allocation of resources such as the awarding of grants to meritorious projects, honors to outstanding students, or journal slots to quality publications. On the supply side, the available budget of grants is awarded to applicants who are evaluated most favorably according to the noisy information available to reviewers. On the demand side, stronger candidates are more likely to obtain grants and thus self-select into applying, given that applications are costly. We establish that if evaluation is perfect, grading on a curve inefficiently discourages even the very best candidates from applying. More generally, when the budget is insufficient to award grants to all applicants, the equilibrium unravels if information is symmetric enough—the paradox of relative evaluation. Leveraging a technique based on the quantile function pioneered by Lehmann, we characterize a broad set of nonmarket allocation rules under which an increase in evaluation noise in a field (or course) raises equilibrium applications in that field, and reduces applications in all other fields. We empirically confirm these comparative statics by exploiting a change in the rule for apportioning the total budget to applications in different fields at the European Research Council, showing that a 1 standard deviation increase in own evaluation noise leads to a 0.4 standard deviation increase in the number of applications and budget share. Moreover, we derive insights for the design of evaluation institutions, particularly regarding the endogenous choice of noise by fields or courses and the optimal aggregation of fields into panels.
Mikhail Golosov, Michael Graber, Magne Mogstad, David Novgorodsky, How Americans Respond to Idiosyncratic and Exogenous Changes in Household Wealth and Unearned Income, The Quarterly Journal of Economics, Volume 139, Issue 2, May 2024, Pages 1321–1395, https://doi.org/10.1093/qje/qjad053
We study how Americans respond to idiosyncratic and exogenous changes in household wealth and unearned income. Our analyses combine administrative data on U.S. lottery winners with an event study design. We first examine individual and household earnings responses to these windfall gains, finding significant and sizable wealth and income effects. On average, an extra $1 of unearned income in a given period reduces household labor earnings by about 50 cents, decreases total labor taxes by 10 cents, and increases consumption expenditure by 60 cents. These effects are heterogeneous across the income distribution, with households in higher quartiles of the income distribution reducing their earnings by a larger amount. Next we examine margins of adjustment other than earnings and, in the course of doing so, address a number of important economic questions about how additional wealth or unearned income affect retirement decisions and labor market dynamics, family formation and dissolution, entrepreneurship and self-employment, and geographic mobility and neighborhood choice. Last, we carefully compare our findings to those reported in existing lottery studies. This comparison reveals that existing U.S. studies substantially underestimate wealth and income effects because they use measures that understate the earnings responses and overstate the after-tax wealth changes associated with lottery wins.
David Autor, Caroline Chin, Anna Salomons, Bryan Seegmiller, New Frontiers: The Origins and Content of New Work, 1940–2018, The Quarterly Journal of Economics, Volume 139, Issue 3, August 2024, Pages 1399–1465, https://doi.org/10.1093/qje/qjae008
We answer three core questions about the hypothesized role of newly emerging job categories (“new work”) in counterbalancing the erosive effect of task-displacing automation on labor demand: what is the substantive content of new work, where does it come from, and what effect does it have on labor demand? We construct a novel database spanning eight decades of new job titles linked to U.S. Census microdata and to patent-based measures of occupations’ exposure to labor-augmenting and labor-automating innovations. The majority of current employment is in new job specialties introduced since 1940, but the locus of new-work creation has shifted from middle-paid production and clerical occupations over 1940–1980 to high-paid professional occupations and secondarily to low-paid services since 1980. New work emerges in response to technological innovations that complement the outputs of occupations and demand shocks that raise occupational demand. Innovations that automate tasks or reduce occupational demand slow new-work emergence. Although the flow of augmentation and automation innovations is positively correlated across occupations, the former boosts occupational labor demand while the latter depresses it. The demand-eroding effects of automation innovations have intensified in the past four decades while the demand-increasing effects of augmentation innovations have not.
Andrea Matranga, The Ant and the Grasshopper: Seasonality and the Invention of Agriculture, The Quarterly Journal of Economics, Volume 139, Issue 3, August 2024, Pages 1467–1504, https://doi.org/10.1093/qje/qjae012
The Neolithic revolution saw the independent development of agriculture among at least seven unconnected hunter-gatherer populations. I propose that the rapid spread of agricultural techniques resulted from increased climatic seasonality causing hunter-gatherers to adopt a sedentary lifestyle and store food for the season of scarcity. Their newfound sedentary lifestyle and storage habits facilitated the invention of agriculture. I present a model and support it with global climate data and Neolithic adoption dates, showing that greater seasonality increased the likelihood of agriculture’s invention and its speed of adoption by neighbors. This study suggests that seasonality patterns played a dominant role in determining our species’ transition to farming.
Simon Jäger, Christopher Roth, Nina Roussille, Benjamin Schoefer, Worker Beliefs About Outside Options, The Quarterly Journal of Economics, Volume 139, Issue 3, August 2024, Pages 1505–1556, https://doi.org/10.1093/qje/qjae001
Standard labor market models assume that workers hold accurate beliefs about the external wage distribution, and hence their outside options with other employers. We test this assumption by comparing German workers’ beliefs about outside options with objective benchmarks. First, we find that workers wrongly anchor their beliefs about outside options on their current wage: workers that would experience a 10% wage change if switching to their outside option only expect a 1% change. Second, workers in low-paying firms underestimate wages elsewhere. Third, in response to information about the wages of similar workers, respondents correct their beliefs about their outside options and change their job search and wage negotiation intentions. Finally, we analyze the consequences of anchoring in a simple equilibrium model. In the model, anchored beliefs keep overly pessimistic workers stuck in low-wage jobs, which gives rise to monopsony power and labor market segmentation.
Nina Roussille, The Role of the Ask Gap in Gender Pay Inequality, The Quarterly Journal of Economics, Volume 139, Issue 3, August 2024, Pages 1557–1610, https://doi.org/10.1093/qje/qjae004
The gender ask gap measures the extent to which women ask for lower salaries than comparable men. This article studies its role in generating wage inequality, using novel data from an online recruitment platform for full-time engineering jobs: Hired.com. To use the platform, job candidates must post an ask salary, stating how much they want to make in their next job. Firms then apply to candidates by offering them a bid salary, solely based on the candidate’s résumé and ask salary. If the candidate is hired, a final salary is recorded. After adjusting for résumé characteristics, the ask gap is 2.9%, the bid gap is 2.2%, and the final offer gap is 1.4%. Further controlling for the ask salary explains the entirety of the residual gender gaps in bid and final salaries. To further provide evidence of the causal effect of the ask salary on the bid salary, I exploit an unanticipated change in how candidates were prompted to provide their ask. For some candidates in mid-2018, the answer box used to solicit the ask salary was changed from an empty field to an entry prefilled with the median bid salary for similar candidates. I find that this change drove the ask, bid, and final offer gaps to zero. In addition, women did not receive fewer bids or final offers than men did due to the change, suggesting they faced little penalty for demanding comparable wages.
E Jason Baron, Joseph J Doyle, Natalia Emanuel, Peter Hull, Joseph Ryan, Discrimination in Multiphase Systems: Evidence from Child Protection, The Quarterly Journal of Economics, Volume 139, Issue 3, August 2024, Pages 1611–1664, https://doi.org/10.1093/qje/qjae007
We develop empirical tools for studying discrimination in multiphase systems and apply them to the setting of foster care placement by child protective services. Leveraging the quasi-random assignment of two sets of decision-makers—initial hotline call screeners and subsequent investigators—we study how unwarranted racial disparities arise and propagate through this system. Using a sample of over 200,000 maltreatment allegations, we find that calls involving Black children are 55% more likely to result in foster care placement than calls involving white children with the same potential for future maltreatment in the home. Call screeners account for up to 19% of this unwarranted disparity, with the remainder due to investigators. Unwarranted disparity is concentrated in cases with potential for future maltreatment, suggesting that white children may be harmed by “underplacement” in high-risk situations.
Ashesh Rambachan, Identifying Prediction Mistakes in Observational Data, The Quarterly Journal of Economics, Volume 139, Issue 3, August 2024, Pages 1665–1711, https://doi.org/10.1093/qje/qjae013
Decision makers, such as doctors, judges, and managers, make consequential choices based on predictions of unknown outcomes. Do these decision makers make systematic prediction mistakes based on the available information? If so, in what ways are their predictions systematically biased? In this article, I characterize conditions under which systematic prediction mistakes can be identified in empirical settings such as hiring, medical diagnosis, and pretrial release. I derive a statistical test for whether the decision maker makes systematic prediction mistakes under these assumptions and provide methods for estimating the ways the decision maker’s predictions are systematically biased. I analyze the pretrial release decisions of judges in New York City, estimating that at least 20% of judges make systematic prediction mistakes about misconduct risk given defendant characteristics. Motivated by this analysis, I estimate the effects of replacing judges with algorithmic decision rules and find that replacing judges with algorithms where systematic prediction mistakes occur dominates the status quo.
Paul Gertler, Brett Green, Catherine Wolfram, Digital Collateral, The Quarterly Journal of Economics, Volume 139, Issue 3, August 2024, Pages 1713–1766, https://doi.org/10.1093/qje/qjae003
A new form of secured lending using “digital collateral” has recently emerged, most prominently in low- and middle-income countries. Digital collateral relies on lockout technology, which allows the lender to temporarily disable the flow value of the collateral to the borrower without physically repossessing it. We explore this new form of credit in a model and a field experiment using school-fee loans digitally secured with a solar home system. Securing a loan with digital collateral drastically reduced default rates (by 19 percentage points) and increased the lender’s rate of return (by 49 percentage points). Using a variant of the Karlan and Zinman (2009) methodology, we decompose the total effect on repayment and find that roughly two-thirds is attributable to moral hazard, and one-third to adverse selection. In addition, access to digitally secured school-fee loans significantly increased school enrollment and school-related expenditures without detrimental effects on households’ balance sheets.
Asaf Bernstein, Peter Koudijs, The Mortgage Piggy Bank: Building Wealth Through Amortization, The Quarterly Journal of Economics, Volume 139, Issue 3, August 2024, Pages 1767–1825, https://doi.org/10.1093/qje/qjae011
In 2013, the Dutch government mandated that new conforming mortgages must fully amortize. Within a difference-in-differences design, we estimate that the marginal wealth accumulation from amortization is close to one, even five years later. Households purchasing after the reform primarily cut consumption and leisure over other savings, leading to a rise in wealth. This holds if we use life events to instrument for the timing of home purchase. Estimates are similar for seemingly unconstrained households and movers, suggesting a broad applicability of our results. Consistent with a simple model, we find lower estimates for households that appear less financially sophisticated or willing to adjust short-term consumption. Mortgage amortization schedules are among the largest savings plans in the world, and our results highlight their critical importance for household wealth building and macroprudential policies.
Martha J Bailey, Thomas Helgerman, Bryan A Stuart, How the 1963 Equal Pay Act and 1964 Civil Rights Act Shaped the Gender Gap in Pay, The Quarterly Journal of Economics, Volume 139, Issue 3, August 2024, Pages 1827–1878, https://doi.org/10.1093/qje/qjae006
In the 1960s, two landmark statutes—the Equal Pay and Civil Rights Acts—targeted the long-standing practice of employment discrimination against U.S. women. For the next 15 years, the gender gap in median earnings among full-time, full-year workers changed little, leading many scholars to conclude that the legislation was ineffectual. This article revisits this conclusion using two research designs, which leverage (i) cross-state variation in preexisting state equal pay laws and (ii) variation in the 1960 gender gap across occupation-industry-state-group cells to capture differences in the legislation's incidence. Both designs suggest that federal antidiscrimination legislation led to striking gains in women's relative wages, which were concentrated among below-median wage earners. These wage gains offset preexisting labor market forces, which worked to depress women's relative pay growth, resulting in the apparent stability of the gender gap at the median and mean in the 1960s and 1970s. The data show little evidence of short-term changes in women's employment but suggest that firms reduced their hiring and promotion of women in the medium to long term. The historical record points to the key role of the Equal Pay Act in driving these changes.
James Feigenbaum, Daniel P Gross, Answering the Call of Automation: How the Labor Market Adjusted to Mechanizing Telephone Operation, The Quarterly Journal of Economics, Volume 139, Issue 3, August 2024, Pages 1879–1939, https://doi.org/10.1093/qje/qjae005
In the early 1900s, telephone operation was among the most common jobs for American women, and telephone operators were ubiquitous. Between 1920 and 1940, AT&T undertook one of the largest automation investments in modern history, replacing operators with mechanical switching technology in over half of the U.S. telephone network. Using variation across U.S. cities in the timing of adoption, we study how this wave of automation affected the labor market for young women. Although automation eliminated most of these jobs, it did not reduce future cohorts’ overall employment: the decline in operators was counteracted by employment growth in middle-skill clerical jobs and lower-skill service jobs, including new categories of work. Using a new genealogy-based census-linking method, we show that incumbent telephone operators were most affected, and a decade later more likely to be in lower-paying occupations or no longer working.
Tasso Adamopoulos, Loren Brandt, Chaoran Chen, Diego Restuccia, Xiaoyun Wei, Land Security and Mobility Frictions, The Quarterly Journal of Economics, Volume 139, Issue 3, August 2024, Pages 1941–1987, https://doi.org/10.1093/qje/qjae010
Frictions that impede the mobility of workers across occupations and space are a prominent feature of developing countries. We disentangle the role of insecure property rights from other labor-mobility frictions for the reallocation of labor from agriculture to nonagriculture and from rural to urban areas. We combine rich household and individual-level panel data from China and an equilibrium quantitative framework featuring sorting of workers across locations and occupations. We explicitly model the farming household and the endogenous decisions of who operates the family farm and who potentially migrates, capturing an additional channel of selection in the household. We find that land insecurity has substantial negative effects on agricultural productivity and structural change, raising the share of rural households operating farms by over 40 percentage points and depressing agricultural productivity by more than 20%. Comparatively, these quantitative effects are as large as those from all residual labor-mobility frictions. We measure a sharp reduction in overall labor-mobility barriers over 2004–2018 in the Chinese economy, all accounted for by improved land security, consistent with reforms covering rural land in China during the period.
Marc Kaufmann, Peter Andre, Botond Kőszegi, Understanding Markets with Socially Responsible Consumers, The Quarterly Journal of Economics, Volume 139, Issue 3, August 2024, Pages 1989–2035, https://doi.org/10.1093/qje/qjae009
Many consumers care about climate change and other externalities associated with their purchases. We analyze the behavior and market effects of such “socially responsible consumers” in three parts. First, we develop a flexible theoretical framework to study competitive equilibria with rational consequentialist consumers. In violation of price taking, equilibrium feedback nontrivially dampens the impact of a person’s consumption on aggregate consumption, undermining the motive to mitigate. This leads to a new type of market failure, where even consumers who fully “internalize the externality” overconsume externality-generating goods. At the same time, socially responsible consumers change the relative effectiveness of taxes, caps, and other policies in lowering the externality. Second, since consumer beliefs about and preferences over their market impacts play a crucial role in our framework, we investigate them empirically via a tailored survey. Consistent with our model, consumers are often consequentialist, and many believe that they have a dampened impact on aggregate consumption. Inconsistent with our model, however, we also find many respondents who expect to have a one-to-one impact on aggregate consumption. Third, therefore, we analyze how such “naive” consumers modify our theoretical conclusions. They consume less than rational consumers in a single-good economy, but may consume more in a multigood economy with cross-market spillovers. A mix of naive and rational consumers may yield the worst outcomes.

Econometrica 2024年1月-8月文章信息(包括摘要):
Abreu, D. and M. Manea (2024). "Bargaining and Exclusion With Multiple Buyers." Econometrica 92(2): 429-465.
A seller trades with q out of n buyers who have valuations a1?≥?a2?≥???≥?an?>?0 via sequential bilateral bargaining. When q?
Agarwal, S., et al. (2024). "Searching for Approval." Econometrica 92(4): 1195-1231.
This paper theoretically and empirically studies the interaction of search and application approval in credit markets. Risky borrowers internalize the probability that their application is rejected and behave as if they had high search costs. Thus, ?overpayment? may be a poor proxy for consumer sophistication since it partly represents rational search in response to rejections. Contrary to standard search models, our model implies (1) endogenous adverse selection through the search and application approval process, (2) a possibly non-monotone or non-decreasing relationship between search and realized interest, default, and application approval rates, and (3) search costs estimated from transaction prices alone are biased. We find support for the model's predictions using a unique data set detailing search behavior of mortgage borrowers. Estimating the model, we find that screening is informative and search is costly. Counterfactual analyses reveal that tightening lending standards and discrimination through application rejection both increase equilibrium interest rates. This increase in realized interest rates is in part due to strategic complementarity in bank rate setting.
Ahn, T., et al. (2024). "Equilibrium Grading Policies With Implications for Female Interest in STEM Courses." Econometrica 92(3): 849-880.
We show that stricter grading policies in STEM courses reduce STEM enrollment, especially for women. We estimate a model of student demand for courses and optimal effort choices given professor grading policies. Grading policies are treated as equilibrium objects that in part depend on student demand for courses. Differences in demand for STEM and non-STEM courses explain much of why STEM classes give lower grades. Restrictions on grading policies that equalize average grades across classes reduce the STEM gender gap and increase overall enrollment in STEM classes.
Almås, I., et al. (2024). "Presidential Address: Economics and Measurement: New Measures to Model Decision Making." Econometrica 92(4): 947-978.
Most empirical work in economics has considered only a narrow set of measures as meaningful and useful to characterize individual behavior, a restriction justified by the difficulties in collecting a wider set. However, this approach often forces the use of strong assumptions to estimate the parameters that inform individual behavior and identify causal links. In this paper, we argue that a more flexible and broader approach to measurement could be extremely useful and allow the estimation of richer and more realistic models that rest on weaker identifying assumptions. We argue that the design of measurement tools should interact with, and depend on, the models economists use. Measurement is not a substitute for rigorous theory, it is an important complement to it, and should be developed in parallel to it. We illustrate these arguments with a model of parental behavior estimated on pilot data that combines conventional measures with novel ones.
Almås, I., et al. (2024). "Reply to: Comment on “Presidential Address: Economics and Measurement: New Measures to Model Decision Making”." Econometrica 92(4): 991-993.Asseyer, A. and R. Weksler (2024). "Certification Design With Common Values." Econometrica 92(3): 651-686.
This paper studies certification design and its implications for information disclosure. Our model features a profit-maximizing certifier and the seller of a good of unknown quality. We allow for common values as the seller's opportunity cost may depend on the quality of the good. We compare certifier-optimal with transparency-maximizing certification design. Certifier-optimal certification design implements the evidence structure of Dye (1985)?a fraction of sellers acquire information while the remaining sellers are uninformed?and results in partial disclosure to the market. A transparency-maximizing regulator prefers a less precise signal, which conveys more information to the market through a higher rate of certification and unraveling (Grossman (1981), Milgrom (1981)) at the disclosure stage.
Baqaee, D. R. and E. Farhi (2024). "Networks, Barriers, and Trade." Econometrica 92(2): 505-541.
We study a flexible class of trade models with international production networks and arbitrary wedge-like distortions like markups, tariffs, or nominal rigidities. We characterize the general equilibrium response of variables to shocks in terms of microeconomic statistics. Our results are useful for decomposing the sources of real GDP and welfare growth, and for computing counterfactuals. Using the same set of microeconomic sufficient statistics, we also characterize societal losses from increases in tariffs and iceberg trade costs and dissect the qualitative and quantitative importance of accounting for disaggregated details. Our results, which can be used to compute approximate and exact counterfactuals, provide an analytical toolbox for studying large-scale trade models and help to bridge the gap between computation and theory.
Bardhi, A. (2024). "Attributes: Selective Learning and Influence." Econometrica 92(2): 311-353.
An agent selectively samples attributes of a complex project so as to influence the decision of a principal. The players disagree about the weighting, or relevance, of attributes. The correlation across attributes is modeled through a Gaussian process, the covariance function of which captures pairwise attribute similarity. The key trade-off in sampling is between the alignment of the players' posterior values for the project and the variability of the principal's decision. Under a natural property of the attribute correlation?the nearest-attribute property (NAP)?each optimal attribute is relevant for some player and at most two optimal attributes are relevant for only one player. We derive comparative statics in the strength of attribute correlation and examine the robustness of our findings to violations of NAP for a tractable class of distance-based covariances. The findings carry testable implications for attribute-based product evaluation and strategic selection of pilot sites.
Basse, G., et al. (2024). "Randomization Tests for Peer Effects in Group Formation Experiments." Econometrica 92(2): 567-590.
Measuring the effect of peers on individuals' outcomes is a challenging problem, in part because individuals often select peers who are similar in both observable and unobservable ways. Group formation experiments avoid this problem by randomly assigning individuals to groups and observing their responses; for example, do first-year students have better grades when they are randomly assigned roommates who have stronger academic backgrounds? In this paper, we propose randomization-based permutation tests for group formation experiments, extending classical Fisher Randomization Tests to this setting. The proposed tests are justified by the randomization itself, require relatively few assumptions, and are exact in finite samples. This approach can also complement existing strategies, such as linear-in-means models, by using a regression coefficient as the test statistic. We apply the proposed tests to two recent group formation experiments.
Bergeron, A., et al. (2024). "The State Capacity Ceiling on Tax Rates: Evidence From Randomized Tax Abatements in the DRC." Econometrica 92(4): 1163-1193.
This paper investigates how tax rates and tax enforcement jointly impact fiscal capacity in low-income countries. We study a policy experiment in the D.R. Congo that randomly assigned 38,028 property owners to the status quo tax rate or to a rate reduction. This variation in tax liabilities reveals that the status quo rate lies above the revenue-maximizing tax rate (RMTR). Reducing rates by about one-third would maximize government revenue by increasing tax compliance. We then exploit two sources of variation in enforcement?randomized enforcement letters and random assignment of tax collectors?to show that the RMTR increases with enforcement. Including an enforcement message on tax letters or replacing tax collectors in the bottom quartile of enforcement capacity with average collectors would raise the RMTR by about 40%. Tax rates and enforcement are thus complementary levers. Jointly optimizing tax rates and enforcement would lead to 10% higher revenue gains than optimizing them independently. These findings provide experimental evidence that low government enforcement capacity sets a binding ceiling on the revenue-maximizing tax rate in some developing countries, thereby demonstrating the value of increasing tax rates in tandem with enforcement to expand fiscal capacity.
Berry, S. T. and P. A. Haile (2024). "Nonparametric Identification of Differentiated Products Demand Using Micro Data." Econometrica 92(4): 1135-1162.
We examine identification of differentiated products demand when one has ?micro data? linking the characteristics and choices of individual consumers. Our model nests standard specifications featuring rich observed and unobserved consumer heterogeneity as well as product/market-level unobservables that introduce the problem of econometric endogeneity. Previous work establishes identification of such models using market-level data and instruments for all prices and quantities. Micro data provides a panel structure that facilitates richer demand specifications and reduces requirements on both the number and types of instrumental variables. We address identification of demand in the standard case in which nonprice product characteristics are assumed exogenous, but also cover identification of demand elasticities and other key features when these product characteristics are endogenous and not instrumented. We discuss implications of these results for applied work.
Bloise, G., et al. (2024). "Do not Blame Bellman: It Is Koopmans' Fault." Econometrica 92(1): 111-140.
We provide a unified approach to stochastic dynamic programming with recursive utility based on an elementary application of Tarski's fixed point theorem. We establish that the exclusive source of multiple values is the presence of multiple recursive utilities consistent with the given aggregator, each yielding a legitimate value of the recursive program. We also present sufficient conditions ensuring a unique value of the recursive program in some circumstances. Overall, acknowledging the unavoidable failure of uniqueness in general, we argue that the greatest fixed point of the Bellman operator should have a privileged position.
Boucher, V., et al. (2024). "Toward a General Theory of Peer Effects." Econometrica 92(2): 543-565.
There is substantial empirical evidence showing that peer effects matter in many activities. The workhorse model in empirical work on peer effects is the linear-in-means (LIM) model, whereby it is assumed that agents are linearly affected by the mean action of their peers. We develop a new general model of peer effects that relaxes the linear assumption of the best-reply functions and the mean peer behavior and that encompasses the spillover, conformist model, and LIM model as special cases. Then, using data on adolescent activities in the United States, we structurally estimate this model. We find that for many activities, individuals do not behave according to the LIM model. We run some counterfactual policies and show that imposing the mean action as an individual social norm is misleading and leads to incorrect policy implications.
Boudreau, L. (2024). "Multinational Enforcement of Labor Law: Experimental Evidence on Strengthening Occupational Safety and Health Committees." Econometrica 92(4): 1269-1308.
Annually, work-related mortality is responsible for 5?7% of all global deaths, and at least 1-in-9 workers experience nonfatal occupational accidents (ILO (2019a,b)). Occupational Safety and Health (OSH) committees are considered the key worker voice institution through which to improve workplace safety and health (ILO (1981)). I present evidence of OSH committees' causal effects on workers and on factories. To do so, I collaborated with 29 multinational apparel buyers that committed to enforce a local mandate for OSH committees on their suppliers in Bangladesh. With the buyers, I implemented a nearly year-long field experiment with 84 supplier factories, randomly enforcing the mandate on half. The buyers' intervention increased compliance with the OSH committee law. Exploiting the experimental variation in OSH committees' strength, I find that stronger OSH committees had small, positive effects on objective measures of safety. These improvements did not come at a cost to workers in terms of wages or employment or to factories in terms of labor productivity. The effects on compliance, safety, and voice were largest for factories with better managerial practices. Factories with worse practices did not improve, and workers in these factories reported lower job satisfaction; this finding suggests complementarity between external enforcement and internal capacity in determining the efficacy of regulation.
Brown, Z. Y. and J. Jeon (2024). "Endogenous Information and Simplifying Insurance Choice." Econometrica 92(3): 881-911.
In markets with complicated products, individuals may choose how much time and effort to spend understanding and comparing alternatives. Focusing on insurance choice, we find evidence consistent with individuals acquiring more information when there are larger consequences from making an uninformed choice. Building on the rational inattention literature, we develop and estimate a parsimonious demand model in which individuals choose how much to research difficult-to-observe characteristics. We use our estimates to evaluate policies that simplify choice. Reducing the number of plans can raise welfare through improved choice as well as savings in information costs. Capping out-of-pocket costs generates larger welfare gains than standard models. The empirical model can be applied to other settings to examine the regulation of complex products.
Caplin, A. (2024). "A Comment on: “Presidential Address: Economics and Measurement: New Measures to Model Decision Making” by Ingvild Almås, Orazio Attanasio, and Pamela Jervis." Econometrica 92(4): 979-985.Che, Y.-K., et al. (2024). "“Near” Weighted Utilitarian Characterizations of Pareto Optima." Econometrica 92(1): 141-165.
We give two characterizations of Pareto optimality via ?near? weighted utilitarian welfare maximization. One characterization sequentially maximizes utilitarian welfare functions using a finite sequence of nonnegative and eventually positive welfare weights. The other maximizes a utilitarian welfare function with a certain class of positive hyperreal weights. The social welfare ordering represented by these ?near? weighted utilitarian welfare criteria is characterized by the standard axioms for weighted utilitarianism under a suitable weakening of the continuity axiom.
Collier, B. and C. Ellis (2024). "A Demand Curve for Disaster Recovery Loans." Econometrica 92(3): 713-748.We estimate and trace a credit demand curve for households that recently experienced damage to their homes from a natural disaster. Our administrative data include over one million applicants to a federal recovery loan program for households. We estimate extensive-margin demand over a large range of interest rates. Our identification strategy exploits 24 natural experiments, leveraging exogenous, time-based variation in the program's offered interest rate. Interest rates meaningfully affect consumer demand throughout the distribution of rates. On average, a 1 percentage point increase in the interest rate reduces loan take-up by 26%. We find a large impact of applicants' credit quality on demand and evidence of monthly payment targeting. Using our estimated demand curve and information on program costs, we find that the program generates an average social surplus of $2900 per borrower.
Deb, S., et al. (2024). "Reply to: Comments on “Walras–Bowley Lecture: Market Power and Wage Inequality”." Econometrica 92(3): 647-650.Deb, S., et al. (2024). "Walras–Bowley Lecture: Market Power and Wage Inequality." Econometrica 92(3): 603-636.
We propose a theory of how market power affects wage inequality. We ask how goods and labor market power jointly determine the level of wages, the skill premium, and wage inequality. We then use detailed microdata from the U.S. Census Bureau between 1997 and 2016 to estimate the parameters of labor supply, technology, and the market structure. We find that a less competitive market structure lowers the average wage of high-skilled workers by 11.3%, and of low-skilled workers by 12.2%, contributes 8.1% to the rise in the skill premium, and accounts for 54.8% of the increase in between-establishment wage variance.
Dilmé, F. (2024). "Sequentially Stable Outcomes." Econometrica 92(4): 1097-1134.
This paper introduces and analyzes sequentially stable outcomes in extensive-form games. An outcome ? is sequentially stable if, for any ε?>?0 and any small enough perturbation of the players' behavior, there is an ε-perturbation of the players' payoffs and a corresponding equilibrium with outcome close to ?. Sequentially stable outcomes exist for all finite games and are outcomes of sequential equilibria. They are closely related to stable sets of equilibria and satisfy versions of forward induction, iterated strict equilibrium dominance, and invariance to simultaneous moves. In signaling games, sequentially stable outcomes pass the standard selection criteria, and when payoffs are generic, they coincide with outcomes of stable sets of equilibria.
Dziewulski, P. and J. K. H. Quah (2024). "Comparative Statics With Linear Objectives: Normality, Complementarity, and Ranking Multi-Prior Beliefs." Econometrica 92(1): 167-200.
We formulate an order over constraint sets , called the parallelogram order, which guarantees that argmin{p???x:x???A} increases in the product order as A increases in the parallelogram order, for any vector . Using this result, we characterize the utility/production functions that lead to normal demand as well as the closely related class of production functions with marginal costs that increase with factor prices. By generalizing the concept of supermodularity, we also characterize the class of production functions for which factors are complements. In the context of decision-making under uncertainty, our new set order leads to natural generalizations of first-order stochastic dominance in multi-prior models.
Feng, F. Z., et al. (2024). "Setbacks, Shutdowns, and Overruns." Econometrica 92(3): 815-847.
We investigate optimal project management in a setting plagued by an indefinite number of setbacks that are discovered en route to project completion. The contractor can cover up delays in progress due to shirking either by making false claims of setbacks or by postponing the reports of real ones. The sponsor optimally induces work and honest reporting via a soft deadline and a reward for completion that specifies a bonus for early delivery. Late-stage setbacks trigger randomization between minimally feasible project extension and (inefficient) cancellation. Because extensions may be granted repeatedly, arbitrarily large overruns in schedule and budget are possible after which the project may still be canceled.
Georgiadis, G., et al. (2024). "Flexible Moral Hazard Problems." Econometrica 92(2): 387-409.
This paper considers a moral hazard problem where the agent can choose any output distribution with a support in a given compact set. The agent's effort-cost is smooth and increasing in first-order stochastic dominance. To analyze this model, we develop a generalized notion of the first-order approach applicable to optimization problems over measures. We demonstrate each output distribution can be implemented and identify those contracts that implement that distribution. These contracts are characterized by a simple first-order condition for each output that equates the agent's marginal cost of changing the implemented distribution around that output with its marginal benefit. Furthermore, the agent's wage is shown to be increasing in output. Finally, we consider the problem of a profit-maximizing principal and provide a first-order characterization of principal-optimal distributions.
Gomez, M. and É. Gouin-Bonenfant (2024). "Wealth Inequality in a Low Rate Environment." Econometrica 92(1): 201-246.
We study the effect of interest rates on wealth inequality. While lower rates decrease the growth rate of rentiers, they also increase the growth rate of entrepreneurs by making it cheaper to raise capital. To understand which effect dominates, we derive a sufficient statistic for the effect of interest rates on the Pareto exponent of the wealth distribution: it depends on the lifetime equity and debt issuance rate of individuals in the right tail of the wealth distribution. We estimate this sufficient statistic using new data on the trajectory of top fortunes in the U.S. Overall, we find that the secular decline in interest rates (or more generally of required rates of returns) can account for about 40% of the rise in Pareto inequality; that is, the degree to which the super rich pulled ahead relative to the rich.
Grant, S., et al. (2024). "A Comment on: “Expected Uncertain Utility”." Econometrica 92(1): 247-256.Haller, A., et al. (2024). "Designing Disability Insurance Reforms: Tightening Eligibility Rules or Reducing Benefits?" Econometrica 92(1): 79-110.
This paper develops a sufficient statistics framework for analyzing the welfare effects of disability insurance (DI). We derive social-optimality conditions for the two main DI policy parameters: (i) eligibility rules and (ii) benefit levels. Applying this framework to two restrictive DI reforms in Austria, we find that tighter DI eligibility rules triggered higher fiscal cost savings and lower insurance losses. Hence, tighter DI eligibility rules dominate DI benefit reductions in scaling back the Austrian DI system.
Hansen, B. E. (2024). "Reply to: Comment on “A Modern Gauss–Markov Theorem”." Econometrica 92(3): 925-928.
This note makes a brief response to Portnoy (2022) and Pötscher and Preinerstorfer (2024), and discusses what instructors should teach about best unbiased estimation.
He, Y., et al. (2024). "Identification and Estimation in Many-to-One Two-Sided Matching Without Transfers." Econometrica 92(3): 749-774.
In a setting of many-to-one two-sided matching with nontransferable utilities, for example, college admissions, we study conditions under which preferences of both sides are identified with data on one single market. Regardless of whether the market is centralized or decentralized, assuming that the observed matching is stable, we show nonparametric identification of preferences of both sides under certain exclusion restrictions. To take our results to the data, we use Monte Carlo simulations to evaluate different estimators, including the ones that are directly constructed from the identification. We find that a parametric Bayesian approach with a Gibbs sampler works well in realistically sized problems. Finally, we illustrate our methodology in decentralized admissions to public and private schools in Chile and conduct a counterfactual analysis of an affirmative action policy.
Herkenhoff, K., et al. (2024). "Production and Learning in Teams." Econometrica 92(2): 467-504.
To what extent is a worker's human capital growth affected by the quality of his coworkers? To answer this question, we develop and estimate a model in which the productivity and the human capital growth of an individual depend on the average human capital of his coworkers. The measured production function is supermodular: The marginal product of a more knowledgeable individual is increasing in the human capital of his coworkers. The measured human capital accumulation function is convex: An individual's human capital growth is increasing in coworkers' human capital only when paired with more knowledgeable coworkers, but independent of coworkers' human capital when paired with less knowledgeable coworkers. Learning from coworkers accounts for two thirds of the stock of human capital accumulated on the job. Technological changes that increase production supermodularity lead to labor market segregation and, by reducing the opportunities for low human capital workers to learn from better coworkers, lead to a decline in aggregate human capital and output.
Herrnstadt, E., et al. (2024). "Drilling Deadlines and Oil and Gas Development." Econometrica 92(1): 29-60.
Oil and gas leases between mineral owners and extraction firms typically specify a date by which the firm must either drill a well or lose the lease. These deadlines are known as primary terms. Using data from the Louisiana shale boom, we first show that well drilling is substantially bunched just before the primary term deadline. This bunching is not necessarily surplus-reducing: using an estimated model of firms' drilling and input choices, we show that primary terms can increase total surplus by countering the effects of leases' royalties, as royalties are a tax on revenue and delay drilling. These benefits are reduced, however, when production outcomes are sensitive to drilling inputs and when drilling one well indefinitely extends the period of time during which additional wells may be drilled. We enrich the model to consider mineral owners' lease offers and find small effects of primary terms on owners' revenue.
Higgins, A. and K. Jochmans (2024). "Bootstrap Inference for Fixed-Effect Models." Econometrica 92(2): 411-427.
The maximum-likelihood estimator of nonlinear panel data models with fixed effects is asymptotically biased under rectangular-array asymptotics. The literature has devoted substantial effort to devising methods that correct for this bias as a means to salvage standard inferential procedures. The chief purpose of this paper is to show that the (recursive, parametric) bootstrap replicates the asymptotic distribution of the (uncorrected) maximum-likelihood estimator and of the likelihood-ratio statistic. This justifies the use of confidence sets and decision rules for hypothesis testing constructed via conventional bootstrap methods. No modification for the presence of bias needs to be made.
Hoffmann, B. and J. P. Rud (2024). "The Unequal Effects of Pollution on Labor Supply." Econometrica 92(4): 1063-1096.
We use high-frequency data on fine particulate matter air pollution (PM 2.5) at the locality level to study the effects of high pollution on daily labor supply decisions in the metropolitan area of Mexico City. We document a negative, non-linear relationship between PM 2.5 and same-day labor supply, with strong effects on days with extremely high pollution levels. On these days, the average worker experiences a reduction of around 7.5% of working hours. Workers partially compensate for lost hours by increasing their labor supply on days that follow high-pollution days. We find that low-income workers reduce their labor supply significantly less than high-income workers. Unequal responses to high pollution along other dimensions (job quality, flexibility, gender) matter, but less than income. We provide suggestive evidence that reductions in labor supply due to high pollution are consistent with avoidance behavior.
Huang, W., et al. (2024). "Learning in Repeated Interactions on Networks." Econometrica 92(1): 1-27.
We study how long-lived, rational agents learn in a social network. In every period, after observing the past actions of his neighbors, each agent receives a private signal, and chooses an action whose payoff depends only on the state. Since equilibrium actions depend on higher-order beliefs, it is difficult to characterize behavior. Nevertheless, we show that regardless of the size and shape of the network, the utility function, and the patience of the agents, the speed of learning in any equilibrium is bounded from above by a constant that only depends on the private signal distribution.
Jiang, Z., et al. (2024). "The U.S. Public Debt Valuation Puzzle." Econometrica 92(4): 1309-1347.
The government budget constraint ties the market value of government debt to the expected present discounted value of fiscal surpluses. We find evidence that U.S. Treasury investors fail to impose this no-arbitrage restriction in the United States. Both cyclical and long-run dynamics of tax revenues and government spending make the surplus claim risky. In a realistic asset pricing model, this risk in surpluses creates a large gap between the market value of debt and its fundamental value, the PDV of surpluses, suggesting that U.S. Treasuries may be overpriced.
Kartik, N., et al. (2024). "Beyond Unbounded Beliefs: How Preferences and Information Interplay in Social Learning." Econometrica 92(4): 1033-1062.
When does society eventually learn the truth, or take the correct action, via observational learning? In a general model of sequential learning over social networks, we identify a simple condition for learning dubbed excludability. Excludability is a joint property of agents' preferences and their information. We develop two classes of preferences and information that jointly satisfy excludability: (i) for a one-dimensional state, preferences with single-crossing differences and a new informational condition, directionally unbounded beliefs; and (ii) for a multi-dimensional state, intermediate preferences and subexponential location-shift information. These applications exemplify that with multiple states, ?unbounded beliefs? is not only unnecessary for learning, but incompatible with familiar informational structures like normal information. Unbounded beliefs demands that a single agent can identify the correct action. Excludability, on the other hand, only requires that a single agent must be able to displace any wrong action, even if she cannot take the correct action.
Kreindler, G. (2024). "Peak-Hour Road Congestion Pricing: Experimental Evidence and Equilibrium Implications." Econometrica 92(4): 1233-1268.
Developing country megacities suffer from severe road traffic congestion, yet the level of congestion is not a direct measure of equilibrium inefficiency. I study the peak-hour traffic congestion equilibrium in Bangalore. To measure travel preferences, I use a model of departure time choice to design a field experiment with congestion pricing policies and implement it using precise GPS data. Commuter responses in the experiment reveal moderate schedule inflexibility and a high value of time. I then show that in Bangalore, traffic density has a moderate and linear impact on travel delay. My policy simulations with endogenous congestion indicate that optimal congestion charges would lead to a small reduction in travel times, and small commuter welfare gains. This result is driven primarily by the shape of the congestion externality. Overall, these results suggest limited commuter welfare benefits from peak-spreading traffic policies in cities like Bangalore.
Lane, G. (2024). "Adapting to Climate Risk With Guaranteed Credit: Evidence From Bangladesh." Econometrica 92(2): 355-386.
Climate change is increasing the frequency of extreme weather events, with low-income countries being disproportionately impacted. However, these countries often face market frictions that hinder their ability to adopt effective adaptation strategies. In this paper, I explore the role of credit market failures in limiting adaptation. To achieve this, I collaborate with a large microfinance institution and offer a randomly selected group of farmers access to guaranteed credit through an ?Emergency Loan? following a negative climate shock. I document three key results. First, farmers who have access to the emergency loan make less costly adaptation choices and are less severely affected when a flood occurs. Second, I find no evidence of adverse spillover effects on households that did not receive the Emergency Loan. Finally, I demonstrate that providing the Emergency Loan is profitable for the microfinance institution, making it a viable tool for the private sector to employ in similar circumstances.
Morris, S., et al. (2024). "Implementation via Information Design in Binary-Action Supermodular Games." Econometrica 92(3): 775-813.
What outcomes can be implemented by the choice of an information structure in binary-action supermodular games? An outcome is partially implementable if it satisfies obedience (Bergemann and Morris (2016)). We characterize when an outcome is smallest equilibrium implementable (induced by the smallest equilibrium). Smallest equilibrium implementation requires a stronger sequential obedience condition: there is a stochastic ordering of players under which players are prepared to switch to the high action even if they think only those before them will switch. We then characterize the optimal outcome induced by an information designer who prefers the high action to be played, but anticipates that the worst (hence smallest) equilibrium will be played. In a potential game, under convexity assumptions on the potential and the designer's objective, it is optimal to choose an outcome where actions are perfectly coordinated (all players choose the same action), with the high action profile played on the largest event where that action profile maximizes the average potential.
Mu, X., et al. (2024). "Monotone Additive Statistics." Econometrica 92(4): 995-1031.
The expectation is an example of a descriptive statistic that is monotone with respect to stochastic dominance, and additive for sums of independent random variables. We provide a complete characterization of such statistics, and explore a number of applications to models of individual and group decision-making. These include a representation of stationary monotone time preferences, extending the work of Fishburn and Rubinstein (1982) to time lotteries. This extension offers a new perspective on risk attitudes toward time, as well as on the aggregation of multiple discount factors. We also offer a novel class of non-expected utility preferences over gambles which satisfy invariance to background risk as well as betweenness, but are versatile enough to capture mixed risk attitudes.
Noack, C. and C. Rothe (2024). "Bias-Aware Inference in Fuzzy Regression Discontinuity Designs." Econometrica 92(3): 687-711.
We propose new confidence sets (CSs) for the regression discontinuity parameter in fuzzy designs. Our CSs are based on local linear regression, and are bias-aware, in the sense that they take possible bias explicitly into account. Their construction shares similarities with that of Anderson?Rubin CSs in exactly identified instrumental variable models, and thereby avoids issues with ?delta method? approximations that underlie most commonly used existing inference methods for fuzzy regression discontinuity analysis. Our CSs are asymptotically equivalent to existing procedures in canonical settings with strong identification and a continuous running variable. However, they are also valid under a wide range of other empirically relevant conditions, such as setups with discrete running variables, donut designs, and weak identification.
Pötscher, B. M. and D. Preinerstorfer (2024). "A Comment on: “A Modern Gauss–Markov Theorem”." Econometrica 92(3): 913-924.
We show that Theorem 4 in Hansen (2022) applies to exactly the same class of estimators as does the classical Aitken theorem. We furthermore point out that Theorems 5?7 in Hansen (2022) contain extra assumptions not present in the classical Gauss?Markov or Aitken theorem, and thus the former theorems do not contain the latter ones as special cases.
Shapiro, J. M. (2024). "A Comment on: “Presidential Address: Economics and Measurement: New Measures to Model Decision Making” by Ingvild Almås, Orazio Attanasio, and Pamela Jervis." Econometrica 92(4): 987-990.Shem-Tov, Y., et al. (2024). "Can Restorative Justice Conferencing Reduce Recidivism? Evidence From the Make-it-Right Program." Econometrica 92(1): 61-78.
This paper studies the effect of a restorative justice intervention targeted at 143 youth ages 13 to 17 facing felony charges of medium severity (e.g., burglary, assault). Eligible youths were randomly assigned to participate in the Make-it-Right (MIR) restorative justice program or a control group where they faced standard criminal prosecution. We estimate the effects of MIR on the likelihood that a youth will be rearrested in the four years following randomization. Assignment to MIR reduces the probability of a rearrest within six months by 19 percentage points, a 44 percent reduction relative to the control group. Moreover, the reduction in recidivism persists even four years after randomization. Thus, our estimates show that restorative justice conferencing can reduce recidivism among youth charged with relatively serious offenses and can be an effective alternative to traditional criminal justice practices.
Van Reenen, J. (2024). "A Comment on: “Walras–Bowley Lecture: Market Power and Wage Inequality” by Shubhdeep Deb, Jan Eeckhout, Aseem Patel, and Lawrence Warren." Econometrica 92(3): 643-646.
A burgeoning literature in labor economics is focused on modeling employer labor market power, generally finding nontrivial estimates of monopsony power. A smaller literature also simultaneously incorporates product market power. Deb, Eeckhout, Patel, and Warren (2024) is an example of applying an oligopoly-oligopsony model to the U.S. labor market, arguing for important effects on wage levels and inequality from rising market power. I support combining IO and labor as a fruitful way of studying wages and business dynamism, but argue for looking more broadly at (i) differential degrees of employer power in labor and product markets; (ii) investigating the dynamic sources of markups (e.g., through innovation), and (iii) considering wage bargaining models, not just wage posting models, which have some starkly different implications for wage setting.
Violante, G. L. (2024). "A Comment on: “Walras–Bowley Lecture: Market Power and Wage Inequality” by Shubhdeep Deb, Jan Eeckhout, Aseem Patel, and Lawrence Warren." Econometrica 92(3): 637-641.

American Economic Review 2024年1月-9月文章信息(包括摘要):
(2) Disentangling Moral Hazard and Adverse Selection
Henrique Castro-Pires, Hector Chade and Jeroen Swinkels
While many real-world principal-agent problems have both moral hazard and adverse selection, existing tools largely analyze only one at a time. Do the insights from the separate analyses survive when the frictions are combined? We develop a simple method—decoupling—to study both problems at once. When decoupling works, everything we know from the separate analyses carries over, but interesting interactions also arise. We provide simple tests for whether decoupling is valid. We develop and numerically implement an algorithm to calculate the decoupled solution and check its validity. We also provide primitives for decoupling to work and analyze several extensions.
Full-Text Access| Supplementary Materials
(3) Universal Basic Income: A Dynamic Assessment
Diego Daruich and Raquel Fernández
Universal basic income (UBI) is an increasingly popular policy proposal, but there is no evidence regarding its longer-term consequences. We find that UBI generates large welfare losses in a general equilibrium model with imperfect capital markets, labor market shocks, and intergenerational linkages via skill formation and transfers. This conclusion is robust to various alternative ways of financing UBI. By using observationally equivalent models that eliminate different sources of endogenous dynamic linkages (equilibrium capital market and parental investment in child skills), we show that the latter are largely responsible for the negative welfare consequences.
Full-Text Access| Supplementary Materials
(4) Retirement Consumption and Pension Design
Jonas Kolsrud, Camille Landais, Daniel Reck and Johannes Spinnewijn
This paper analyzes consumption to evaluate the distributional effects of pension reforms. Using Swedish administrative data, we show that on average, workers who retire earlier consume less while retired and experience larger drops in consumption around retirement. Interpreted via a theoretical model, these findings imply that reforms incentivizing later retirement incur a substantial consumption smoothing cost. Turning to other features of pension policy, we find that reforms that redistribute based on early-career labor supply would have opposite-signed redistributive effects, while differentiating on wealth may help to target pension benefits toward those who are vulnerable to larger drops in consumption around retirement.
Full-Text Access| Supplementary Materials
(5) The Ends of 27 Big Depressions
Martin Ellison, Sang Seok Lee and Kevin Hjortshøj O'Rourke
How did countries recover from the Great Depression? In this paper, we explore the argument that leaving the gold standard helped by boosting inflationary expectations, lowering real interest rates, and stimulating interest-sensitive expenditures. We do so for a sample of 27 countries, using modern nowcasting methods and a new dataset containing more than 230,000 monthly and quarterly observations for over 1,500 variables. In those cases where the departure from gold happened on well-defined dates, inflationary expectations clearly rose in the wake of departure. Instrumental variable, difference-in-difference, and synthetic matching techniques suggest that the relationship is causal.
Full-Text Access| Supplementary Materials
(6) The Economic Impact of Depression Treatment in India: Evidence from Community-Based Provision of Pharmacotherapy
Manuela Angelucci and Daniel Bennett
This study evaluates the impact of depression treatment on economic behavior in Karnataka, India. We cross-randomize pharmacotherapy and livelihoods assistance among 1,000 depressed adults and evaluate impacts on depression severity, socioeconomic outcomes, and several potential pathways. When combined, the interventions reduce depression severity, with benefits that persist after treatment concludes. Pharmacotherapy alone has a weaker effect that is only marginally significant and dissipates sooner. Depression treatment does not significantly increase earnings, consumption, or human capital investment in children.
Full-Text Access| Supplementary Materials
(7) Market Power and Innovation in the Intangible Economy
Maarten De Ridder
This paper offers a unified explanation for the slowdown of productivity growth, the decline in business dynamism, and the rise of market power. Using a quantitative framework, I show that the rise of intangible inputs, such as software, can explain these trends. Intangibles reduce marginal costs and raise fixed costs, which gives firms with high-intangible adoption a competitive advantage, in turn deterring other firms from entering. I structurally estimate the model on French and US micro data. After initially boosting productivity, the rise of intangibles causes a decline in productivity growth, consistent with the empirical trends observed since the mid-1990s.
Full-Text Access| Supplementary Materials
(8) Propagation and Insurance in Village Networks
Cynthia Kinnan, Krislert Samphantharak, Robert Townsend and Diego Vera-Cossio
Firms in developing countries are embedded in supply chains and labor networks. These linkages may propagate or attenuate shocks. Using panel data from Thai villages, we document three facts: as households facing idiosyncratic shocks adjust their production, these shocks propagate to other households on both the production and consumption sides; propagation is greater via labor than supply chain links; and shocks in denser networks and to more central households propagate more, while access to formal or informal insurance reduces propagation. Social benefits from expanding safety nets may be higher than private benefits.
Full-Text Access| Supplementary Materials
(9) Competing to Commit: Markets with Rational Inattention
Carlo M. Cusumano, Francesco Fabbri and Ferdinand Pieroth
Two homogeneous-good firms compete for a consumer's unitary demand. The consumer is rationally inattentive and pays entropy costs to process information about firms' offers. Compared to a collusion benchmark, competition produces two effects. As in standard models, competition puts downward pressure on prices. But, additionally, an attention effect arises: the consumer engages in trade more often. This alleviates the commitment problem that firms have when facing inattentive consumers and increases trade efficiency. For high enough attention costs, the attention effect dominates the effect on prices: firms' profits are higher under competition than under collusion.
(2) Distinguishing Common Ratio Preferences from Common Ratio Effects Using Paired Valuation Tasks

Christina McGranaghan, Kirby Nielsen, Ted O'Donoghue, Jason Somerville and Charles D. Sprenger
Without strong assumptions about how noise manifests in choices, we can infer little from existing empirical observations of the common ratio effect (CRE) about whether there exists an underlying common ratio preference (CRP). We propose to solve this inferential challenge using paired valuations, which yield valid inference under common assumptions. Using this approach in an online experiment with 900 participants, we find no evidence of a systematic CRP. To reconcile our findings with existing evidence, we present the same participants with paired choice tasks and demonstrate how noise can generate a CRE even for individuals without an associated CRP.
Full-Text Access| Supplementary Materials
(3) The Immigrant Next Door
Leonardo Bursztyn, Thomas Chaney, Tarek A. Hassan and Aakaash Rao
We study how decades-long exposure to individuals of a give foreign descent shapes natives' attitudes and behavior toward that group. Using individualized donations data, we show that long-term exposure to a given foreign ancestry leads to more generous behavior specifically toward that group's ancestral country. Focusing on exposure to Arab Muslims to examine mechanisms, we show that long-term exposure (i) decreases explicit and implicit prejudice against Arab Muslims, (ii) reduces support for policies and political candidates hostile toward Arab Muslims, (iii) increases charitable donations to Arab countries, (iv) leads to more personal contact with Arab Muslims, and (v) increases knowledge of Arab Muslims and Islam.
Full-Text Access| Supplementary Materials
(4) The Nurture of Nature and the Nature of Nurture: How Genes and Investments Interact in the Formation of Skills
Mikkel Aagaard Houmark, Victor Ronda and Michael Rosholm
This paper studies the interplay between genetics and family investments in the process of skill formation. We model and estimate the joint evolution of skills and parental investments throughout early childhood. We document three genetic mechanisms: the direct effect of child genes on skills, the indirect effect of child genes via parental investments, and family genetic influences captured by parental genes. We show that genetic effects are dynamic, increase over time, and operate via environmental channels. Our paper highlights the value of integrating biological and social perspectives into a single unified framework.
Full-Text Access| Supplementary Materials
(5) Prolonged Learning and Hasty Stopping: The Wald Problem with Ambiguity
Sarah Auster, Yeon-Koo Che and Konrad Mierendorff
This paper studies sequential information acquisition by an ambiguity-averse decision-maker (DM), who decides how long to collect information before taking an irreversible action. The agent optimizes against the worst-case belief and updates prior by prior. We show that the consideration of ambiguity gives rise to rich dynamics: compared to the Bayesian DM, the DM here tends to experiment excessively when facing modest uncertainty and, to counteract it, may stop experimenting prematurely when facing high uncertainty. In the latter case, the DM's stopping rule is nonmonotonic in beliefs and features randomized stopping.
Full-Text Access| Supplementary Materials
(6) Gender Differences in Medical Evaluations: Evidence from Randomly Assigned Doctors
Marika Cabral and Marcus Dillender
Little is known about what drives gender disparities in health care and related social insurance benefits. Using data and variation from the Texas workers' compensation program, we study the impact of gender match between doctors and patients on medical evaluations and associated disability benefits. Compared to differences among their male patient counterparts, female patients randomly assigned a female doctor rather than a male doctor are 5.2 percent more likely to be evaluated as disabled and receive 8.6 percent more subsequent cash benefits on average. There is no analogous gender-match effect for male patients. Our estimates indicate that increasing the share of female patients evaluated by female doctors may substantially shrink gender gaps in medical evaluations and associated outcomes.
Full-Text Access| Supplementary Materials
(7) Pricing Power in Advertising Markets: Theory and Evidence
Matthew Gentzkow, Jesse M. Shapiro, Frank Yang and Ali Yurukoglu
Existing theories of media competition imply that advertisers will pay a lower price in equilibrium to reach consumers who multi-home across competing outlets. We generalize and extend this theoretical result and test it using data from television and social media advertising. We find that the model is a good match, qualitatively and quantitatively, to variation in advertising prices across demographic groups, outlets, platforms, and over time. We use the model to quantify the effects of competition within and across platforms.
Full-Text Access| Supplementary Materials
(8) Asset-Based Microfinance for Microenterprises: Evidence from Pakistan
Faisal Bari ⓡ Kashif Malik ⓡ Muhammad Meki ⓡ Simon Quinn
We run a field experiment offering graduated microcredit clients the opportunity to finance a business asset worth four times their usual borrowing limit. We implement this using a hire-purchase contract; our control group is offered a zero interest loan at the usual borrowing limit. We find large, significant, and persistent effects: treated microenterprise owners run larger businesses with higher profits; consequently, household consumption increases, particularly on food and children's education. A dynamic structural model with nonconvex capital adjustment costs rationalizes our results and allows counterfactual analysis; this highlights the potential for welfare improvements through large capital injections that are financially sustainable.
Full-Text Access| Supplementary Materials
(9) Old Age Risks, Consumption, and Insurance
Richard Blundell, Margherita Borella, Jeanne Commault and Mariacristina De Nardi
In the United States, after age 65, households face income and health risks, and a large fraction of these risks are transitory. While consumption significantly responds to transitory income shocks, out-of-pocket medical expenses do not. In contrast, both consumption and out-of-pocket medical expenses respond to transitory health shocks. Thus, most US elderly keep their out-of-pocket medical expenses close to a satiation point that varies with health. Consumption responds to health shocks mostly because adverse health shocks reduce the marginal utility of consumption. The effect of health on marginal utility changes the optimal transfers due to health shocks.
(2) The Economics of the Public Option: Evidence from Local Pharmaceutical Markets

Juan Pablo Atal, José Ignacio Cuesta, Felipe González and Cristóbal Otero
We study the effects of competition by state-owned firms, leveraging the decentralized entry of public pharmacies to local markets in Chile. Public pharmacies sell the same drugs at a third of private pharmacy prices, because of stronger upstream bargaining and market power in the private sector, but are of lower quality. Public pharmacies induced market segmentation and price increases in the private sector, which benefited the switchers to the public option but harmed the stayers. The countrywide entry of public pharmacies would reduce yearly consumer drug expenditure by 1.6 percent.
Full-Text Access| Supplementary Materials
(3) The Effect of Macroeconomic Uncertainty on Household Spending
Olivier Coibion, Dimitris Georgarakos, Yuriy Gorodnichenko, Geoff Kenny and Michael Weber
We use randomized treatments that provide different types of information about the first and/or second moments of future economic growth to generate exogenous changes in the perceived macroeconomic uncertainty of treated households. The effects on their spending decisions relative to an untreated control group are measured in follow-up surveys. Our results indicate that, after taking into account first moments, higher macroeconomic uncertainty induces households to significantly and persistently reduce their total monthly spending in subsequent months. Changes in spending are broad based across spending categories and apply to larger durable good purchases as well.
Full-Text Access| Supplementary Materials
(4) Optimal Inference for Spot Regressions
Tim Bollerslev, Jia Li and Yuexuan Ren
Betas from return regressions are commonly used to measure systematic financial market risks. "Good" beta measurements are essential for a range of empirical inquiries in finance and macroeconomics. We introduce a novel econometric framework for the nonparametric estimation of time-varying betas with high-frequency data. The "local Gaussian" property of the generic continuous-time benchmark model enables optimal "finite-sample" inference in a well-defined sense. It also affords more reliable inference in empirically realistic settings compared to conventional large-sample approaches. Two applications pertaining to the tracking performance of leveraged ETFs and an intraday event study illustrate the practical usefulness of the new procedures.
Full-Text Access| Supplementary Materials
(5) The Comparative Statics of Sorting
Axel Anderson and Lones Smith
We create a general and tractable theory of increasing sorting in pairwise matching models with monetary transfers. The positive quadrant dependence partial order subsumes Becker (1973) as the extreme cases with most and least sorting and implies increasing regression coefficients. Our theory turns on synergy—the cross-partial difference or derivative of match production. This reflects basic economic forces: diminishing returns, technological convexity, insurance, and learning dynamics. We prove sorting increases if match synergy globally increases, and is cross-sectionally monotone or single crossing. We use our results to derive sorting predictions in major economics sorting papers and in new applications.
Full-Text Access| Supplementary Materials
(6) Mental Models and Learning: The Case of Base-Rate Neglect
Ignacio Esponda, Emanuel Vespa and Sevgi Yuksel
We experimentally document persistence of suboptimal behavior despite ample opportunities to learn from feedback in a canonical updating problem where people suffer from base-rate neglect. Our results provide insights on the mechanisms hindering learning from feedback. Importantly, our results suggest mistakes are more likely to be persistent when they are driven by incorrect mental models that miss or misrepresent important aspects of the environment. Such models induce confidence in initial answers, limiting engagement with and learning from feedback. We substantiate these insights in an alternative scenario where individuals involved in a voting problem overlook the importance of being pivotal.
Full-Text Access| Supplementary Materials
(7) Hub-and-Spoke Cartels: Theory and Evidence from the Grocery Industry
Robert Clark, Ig Horstmann and Jean-François Houde
Numerous recently uncovered cartels operated along the supply chain, with firms at one end facilitating collusion at the other—hub-and-spoke arrangements. These cartels are hard to rationalize because they induce double marginalization and higher costs. We examine Canada's alleged bread cartel and provide the first comprehensive analysis of hub-and-spoke collusion. Using court documents and pricing data, we make three contributions: (i) we show that collusion was effective, increasing inflation by about 50 percent; (ii) we provide evidence that collusion existed at both ends of the supply chain; and (iii) we develop a model explaining why this form of collusion arose.
Full-Text Access| Supplementary Materials
(8) Does the Squeaky Wheel Get More Grease? The Direct and Indirect Effects of Citizen Participation on Environmental Governance in China
Mark T. Buntaine, Michael Greenstone, Guojun He, Mengdi Liu, Shaoda Wang and Bing Zhang
We conducted a nationwide field experiment in China to evaluate the direct and indirect impacts of assigning firms to public or private citizen appeals when they violate pollution standards. There are three main findings. First, public appeals to the regulator through social media substantially reduce violations and pollution emissions, while private appeals cause more modest environmental improvements. Second, public appeals appear to tilt regulators' focus away from facilitating economic growth and toward avoiding pollution-induced public unrest. Third, pollution reductions by treated firms are not offset by control firms, based on randomly varying the proportion of treated firms at the prefecture level.
Full-Text Access| Supplementary Materials
| (9) The Gender Gap in Confidence: Expected but Not Accounted For || Christine L. Exley and Kirby Nielsen                         || We investigate how the gender gap in confidence affects the views that evaluators (e.g., employers) hold about men and women. We find the confidence gap is contagious, causing evaluators to form overly pessimistic beliefs about women. This result arises even though the confidence gap is expected and even though the confidence gap shouldn't be contagious if evaluators are Bayesian. Only an intervention that facilitates Bayesian updating proves (somewhat) effective. Additional results highlight how similar findings follow even when there is no room for discriminatory motives or differences in priors because evaluators are asked about arbitrary, rather than gender-specific, groups. |
(2) Is Journalistic Truth Dead? Measuring How Informed Voters Are about Political News

Charles Angelucci and Andrea Prat
To investigate general patterns in news information in the United States, we combine a protocol for identifying major political news stories, 11 monthly surveys with 15,000 participants, and a model of news discernment. When confronted with a true and a fake news story, 47 percent of subjects confidently choose the true story, 3 percent confidently choose the fake story, and the remaining half are uncertain. Socioeconomic differences are associated with large variations in the probability of selecting the true news story. Partisan congruence between an individual and a news story matters, but its impact is up to an order of magnitude smaller.
Full-Text Access| Supplementary Materials
(3) Anticipatory Anxiety and Wishful Thinking
Jan B. Engelmann, Maël Lebreton, Nahuel A. Salem-Garcia, Peter Schwardmann and Joël J. van der Weele
Across five experiments (N = 1,714), we test whether people engage in wishful thinking to alleviate anxiety about adverse future outcomes. Participants perform pattern recognition tasks in which some patterns may result in an electric shock or a monetary loss. Diagnostic of wishful thinking, participants are less likely to correctly identify patterns that are associated with a shock or loss. Wishful thinking is more pronounced under more ambiguous signals and only reduced by higher accuracy incentives when participants' cognitive effort reduces ambiguity. Wishful thinking disappears in the domain of monetary gains, indicating that negative emotions are important drivers of the phenomenon.
Full-Text Access| Supplementary Materials
(4) Motivated Errors
Christine L. Exley and Judd B. Kessler
Myriad environments allow for the possibility of confusion. Agents may appeal to such confusion—or the possibility of making an honest mistake—to justify their behavior. In three sets of experiments involving thousands of subjects, we document evidence of such motivated errors. We document this evidence in a simple environment in which the scope for errors is small and in more complex environments in which subjects display correlation neglect and an anchoring bias.
Full-Text Access| Supplementary Materials
(5) When Tariffs Disrupt Global Supply Chains
Gene M. Grossman, Elhanan Helpman and Stephen J. Redding
We study unanticipated tariffs in a setting with firm-to-firm supply relationships. Firms conduct costly searches and negotiate with potential suppliers that pass a reservation level of match productivity. Global supply chains form in anticipation of free trade. Then, the home government surprises with an input tariff. This can lead to renegotiation with initial suppliers or search for replacements. Calibrating the model's parameters to match initial import shares and the estimated responses to the US tariffs imposed on China, we find an overall welfare loss of 0.12 percent of GDP, with substantial contributions from changes in input sourcing and search costs.
Full-Text Access| Supplementary Materials
(6) Local Productivity Spillovers
Nathaniel Baum-Snow, Nicolas Gendron-Carrier and Ronni Pavan
Using Canadian administrative data, this paper presents evidence of revenue and productivity spillovers across firms at fine spatial scales. Accounting for the endogenous sorting of firms across space, we estimate an average elasticity of firm revenue and productivity with respect to the average quality of other firms within 75 meters of 0.024. We find scant evidence that the average firm benefits from being surrounded by a greater amount of economic activity at this spatial scale. Sorting of higher-quality firms into more productive locations and higher average and aggregate quality peer groups is salient in the data.
Full-Text Access| Supplementary Materials
(7) Redesigning the US Army's Branching Process: A Case Study in Minimalist Market Design
Kyle Greenberg, Parag A. Pathak and Tayfun Sönmez
We present a proof-of-concept for minimalist market design (Sönmez 2023) as an effective methodology to enhance an institution based on stakeholders' desiderata with minimal interference. Four objectives—respecting merit, increasing retention, aligning talent, and enhancing trust—guided reforms to the US Army's centralized branching process of cadets to military specialties since 2006. USMA's mechanism for the class of 2020 exacerbated challenges in implementing these objectives. Formulating the Army's desiderata as rigorous axioms, we analyze their implications. Under our minimalist approach to institution redesign, the Army's objectives uniquely identify a branching mechanism. Our design is now adopted at USMA and ROTC.
Full-Text Access| Supplementary Materials
(8) The Political Development Cycle: The Right and the Left in People's Republic of China from 1953
Anton Cheremukhin, Mikhail Golosov, Sergei Guriev and Aleh Tsyvinski
We quantify the effects of the political development cycle—the fluctuations between the Left (Maoist) and the Right (pragmatist) development policies—on growth and structural transformation of China in 1953–1978. The left policies prioritized structural transformation toward nonagricultural production and consumption at the expense of agricultural development. The right policies prioritized agricultural consumption through slower structural transformation. The imperfect implementation of these policies led to large welfare costs of the political development cycle in a distorted economy undergoing a structural change.
Full-Text Access| Supplementary Materials
(9) The Opportunity Cost of Debt Aversion
Alejandro Martínez-Marquina and Mike Shi
We provide evidence of the existence of debt aversion and its negative implications for financial decisions. In a new experimental design where subjects are assigned debt randomly, we quantify the opportunity cost of subjects' debt-biased decisions. One-third of our participants neglect high returns and focus instead on debt repayments. In addition, borrowing to invest is 50 percent less likely when it leads to indebtedness. On average, participants perceive $1 less in debt as equivalent to $1.03 in savings. Hence, a debt-averse agent will undertake a 10 percent guaranteed investment only if the cost of borrowing does not exceed 6.80 percent.
Full-Text Access| Supplementary Materials
(10) Nurturing Childhood Curiosity to Enhance Learning: Evidence from a Randomized Pedagogical Intervention
Sule Alan and Ipek Mumcu
We evaluate a pedagogical intervention aimed at improving learning in elementary school children by fostering their curiosity. We test the effectiveness of the pedagogy using achievement scores and a novel measure of curiosity. The latter involves creating a sense of information deprivation and quantifying the urge to acquire information and retention ability. The intervention increases curiosity, knowledge retention, and science test scores, with the effects persisting into middle school years. It also leads to more information sharing and peer learning in the classroom. The evidence can help design better pedagogical tools to increase pupil engagement and the quality of learning.
(2) Shocks, Frictions, and Inequality in US Business Cycles

Christian Bayer, Benjamin Born and Ralph Luetticke
We show how a heterogeneous agent New Keynesian (HANK) model with incomplete markets and portfolio choice can be estimated in state space using a Bayesian approach. To render estimation feasible, the structure of the economy can be exploited and the dimensionality of the model automatically reduced based on the Bayesian priors. We apply this approach to analyze how much inequality matters for the business cycle and vice versa. Even when the model is estimated on aggregate data alone and with a set of shocks and frictions designed to match aggregate data, it broadly reproduces observed US inequality dynamics.
Full-Text Access| Supplementary Materials
(3) Household Labor Supply and the Value of Social Security Survivors Benefits
David Coyne, Itzik Fadlon, Shanthi P. Ramnath and Patricia K. Tong
We combine quasi-experimental variation in spousal death and age eligibility for survivors benefits using US tax records to study the effects on American households' labor supply and the design of social security's survivors insurance. Benefit eligibility at the exact age of 60 induces sharp reductions in the labor supply of newly widowed households, highlighting the value of survivors benefits and the liquidity they provide following the shock. Among eligible widows, the spousal death event induces no increases in labor supply, suggesting little residual need to self-insure. Using theory, we underscore the program's protective insurance role and its high valuation among survivors.
Full-Text Access| Supplementary Materials
(4) Creating Moves to Opportunity: Experimental Evidence on Barriers to Neighborhood Choice
Peter Bergman, Raj Chetty, Stefanie DeLuca, Nathaniel Hendren, Lawrence F. Katz and Christopher Palmer
Low-income families often live in low-upward-mobility neighborhoods. We study why by using a randomized trial with housing voucher recipients that provided information, financial support, and customized search assistance to move to high-opportunity neighborhoods. The treatment increased the fraction moving to high-upward-mobility areas from 15 to 53 percent. A second trial reveals this treatment effect is driven primarily by customized search assistance. Qualitative interviews show that the intervention relaxed bandwidth constraints and addressed family-specific needs. Our findings imply many low-income families do not have strong preferences to stay in low-opportunity areas and that barriers in housing search significantly increase residential segregation by income.
Full-Text Access| Supplementary Materials
(5) From Fog to Smog: The Value of Pollution Information
Panle Jia Barwick, Shanjun Li, Liguo Lin and Eric Yongchen Zou
In 2013, China launched a landmark program to monitor air quality and disclose real-time data, significantly increasing the public's access to and awareness of pollution information. The program triggered cascading behavioral changes such as stronger avoidance of outdoor pollution exposure and increased spending on protective products. These behavioral responses mitigated the mortality impact of air pollution. Conservative estimates indicate that the program's health benefits outweigh the costs by an order of magnitude. The findings highlight the benefits of improving public access to pollution information in developing countries which often experience severe air pollution but lack pollution data collection and dissemination.
Full-Text Access| Supplementary Materials
(6) Urban Public Works in Spatial Equilibrium: Experimental Evidence from Ethiopia
Simon Franklin, Clément Imbert, Girum Abebe and Carolina Mejia-Mantilla
This paper evaluates a large urban public works program randomly rolled out across neighborhoods of Addis Ababa, Ethiopia. We find the program increased public employment and reduced private labor supply among beneficiaries and improved local amenities in treated locations. We then combine a spatial equilibrium model and unique commuting data to estimate the spillover effects of the program on private sector wages across neighborhoods: under full program rollout, wages increased by 18.6 percent. Using our model, we show that welfare gains to the poor are six times larger when we include the indirect effects on private wages and local amenities.
Full-Text Access| Supplementary Materials
(7) Resisting Social Pressure in the Household Using Mobile Money: Experimental Evidence on Microenterprise Investment in Uganda
Emma Riley
I examine whether changing the form of disbursement of a microfinance loan enables female microfinance borrowers to overcome intra-household sharing pressure and grow their businesses. Using a field experiment with 3,000 borrowers in Uganda, I compare the disbursement of a loan as cash to disbursement onto a digital account. After 8 months, women who received their microfinance loan on the digital account had 11 percent higher (US$70) business capital and 15 percent higher (US$18) profits compared to those who received their loan as cash. Impacts were greatest for women who experienced pressure to share money with others in the household at baseline.
Full-Text Access| Supplementary Materials
(8) A Framework for Economic Growth with Capital-Embodied Technical Change
Benjamin F. Jones and Xiaojie Liu
Technological advance is often embodied in capital inputs, like computers, airplanes, and robots. This paper builds a framework where capital inputs advance through (i) increased automation and (ii) increased productivity. The interplay of these two innovation dimensions can produce balanced growth, satisfying the Uzawa Growth Theorem even though technological progress is capital-embodied. The framework can further address structural transformation, general-purpose technologies, the limited macroeconomic impact of computing, and declining productivity growth and labor shares. Overall, this tractable framework can help resolve puzzling tensions between micro-level observations of innovation and balanced growth while providing new perspectives on numerous macroeconomic phenomena.
Full-Text Access| Supplementary Materials
(9) On Binscatter
Matias D. Cattaneo, Richard K. Crump, Max H. Farrell and Yingjie Feng
Binscatter is a popular method for visualizing bivariate relationships and conducting informal specification testing. We study the properties of this method formally and develop enhanced visualization and econometric binscatter tools. These include estimating conditional means with optimal binning and quantifying uncertainty. We also highlight a methodological problem related to covariate adjustment that can yield incorrect conclusions. We revisit two applications using our methodology and find substantially different results relative to those obtained using prior informal binscatter methods. General purpose software in Python, R, and Stata is provided. Our technical work is of independent interest for the nonparametric partition-based estimation literature.
(2) Nobel Lecture: An Evolving Economic Force

Claudia Goldin
Full-Text Access | Supplementary Materials  
(3) Local Economic and Political Effects of Trade Deals: Evidence from NAFTA
Jiwon Choi, Ilyana Kuziemko, Ebonya Washington, and Gavin Wright
Why have white, less-educated voters left the Democratic Party? We highlight the role of the 1994 North American Free Trade Agreement (NAFTA). In event-study analysis, we demonstrate that counties whose 1990 employment depended on industries vulnerable to NAFTA suffered large and persistent employment losses after its implementation. Voters in these counties (and protectionist voters regardless of geography) turned away from the party of President Clinton, who promoted the agreement. This shift is larger for whites (especially men and those without a college degree) and social conservatives, suggesting that racial identity and social-issue positions mediate reactions to economic policies.
Full-Text Access | Supplementary Materials  
(4) Status Externalities in Education and Low Birth Rates in Korea
Seongeun Kim, Michèle Tertilt, and Minchul Yum
South Koreans appear to be preoccupied with their offspring's education and also have the lowest total fertility rate in the world. We propose a novel theory with status externalities and endogenous fertility connecting these facts, motivated by novel empirical evidence on private education spillovers. Using a quantitative model calibrated to Korea, we find that fertility would be 28 percent higher without the externality. We explore the effects of government policy: a pro-natal transfer or an education tax can increase fertility and reduce education spending. An education tax of 22 percent together with moderate pro-natal transfers maximizes the current generation's welfare.
Full-Text Access | Supplementary Materials  
(5) Welfare Comparisons for Biased Learning
Mira Frick, Ryota Iijima, and Yuhta Ishii
We study robust welfare comparisons of learning biases (misspecified Bayesian and some forms of non-Bayesian updating). Given a true signal distribution, we deem one bias more harmful than another if it yields lower objective expected payoffs in all decision problems. We characterize this ranking in static and dynamic settings. While the static characterization compares posteriors signal by signal, the dynamic characterization employs an "efficiency index" measuring how fast beliefs converge. We quantify and compare the severity of several well-documented biases. We also highlight disagreements between the static and dynamic rankings, and that some "large" biases dynamically outperform other "vanishingly small" biases.
Full-Text Access | Supplementary Materials  
(6) The Flight to Safety and International Risk Sharing
Rohan Kekre and Moritz Lenel
We study a business cycle model of the international monetary system featuring a time varying demand for safe dollar bonds, greater risk-bearing capacity in the United States than the rest of the world, and nominal rigidities. A flight to safety generates a dollar appreciation and decline in global output. Dollar bonds thus command a negative risk premium, and the United States holds a levered portfolio of capital finances in dollars. We quantify the effects of safety shocks and heterogeneity in risk-bearing capacity for global macroeconomic volatility, US external adjustment, and policy transmission, as of dollar swap lines.
Full-Text Access | Supplementary Materials  
(7) Long-Run Effects of Incentivizing Work after Childbirth
Elira Kuka and Na'ama Shenhav
This paper identifies the impact of increasing post-childbirth work incentives on mothers' long-run careers. We exploit variation in work incentives across mothers based on the timing of a first birth and eligibility for the 1993 expansion of the Earned Income Tax Credit. Ten to nineteen years after a first birth, single mothers who were exposed to the expansion immediately after birth ("early"), rather than 3–6 years later ("late"), have 0.62 more years of work experience and 4.2 percent higher earnings conditional on working. We show that higher earnings are primarily explained by improved wages due to greater work experience.
Full-Text Access | Supplementary Materials  
(8) Task-Based Discrimination
Erik Hurst, Yona Rubinstein, and Kazuatsu Shimizu
We develop a task-based model of occupational sorting to identify and quantify the effect of discrimination, racial skill gaps, and aggregate task prices on Black-White differences in labor market outcomes over time. At the heart of our framework is the idea that the size and nature of racial barriers faced by Black workers vary by the task requirements of each job. We define a new task that measures the extent to which individuals interact with others as part of their job. We show that this measure is a good proxy for the extent of discrimination in the economy.
Full-Text Access | Supplementary Materials  
(9) Information Technology and Returns to Scale
Danial Lashkari, Arthur Bauer, and Jocelyn Boussard
What are the implications of the dramatic fall in IT prices for aggregate technology? When firm-level technologies are continuously differentiable, a factor price shock leads to (i) a substitution between factors and/or (ii) an endogenous response of returns to scale. The second channel is governed by the output elasticity of relative factor demand. Using detailed firm-level data from France, we estimate this elasticity to be positive for IT factor demand. A quantitative exercise accounting for both technological channels shows that falling IT prices can explain much of the changes in concentration and the composition of aggregate labor share in France.
Full-Text Access | Supplementary Materials  
(10) Breaking Gender Barriers: Experimental Evidence on Men in Pink-Collar Jobs
Alexia Delfino
I investigate men's limited entry into female-dominated sectors through a large-scale field experiment. The design exogenously varies recruitment messages by showing photographs of current workers (male or female) and providing information on the share of workers who received high evaluations in the past (higher or lower). A male photograph has no impact on men's applications, but informing about a lower share of high evaluations encourages men to apply and enables the employer to hire and retain more talented men. The impact of this informational intervention remains positive for the employer also accounting for its effects on female applicants and hires.
Full-Text Access | Supplementary Materials  
(11) Financial Access and Labor Market Outcomes: Evidence from Credit Lotteries
Bernardus Van Doornik, Armando Gomes, David Schoenherr, and Janis Skrastins
We assess the employment and income effects of access to credit dedicated to investment in individual mobility by exploiting time-series variation in access to credit through lotteries for participants in a group-lending mechanism in Brazil. We find that access to credit for investment in individual mobility increases formal employment rates and salaries, yielding an annual rate of return of 12 to 15 percent. Consistent with a geographically broader job search, individuals transition to jobs farther from home and public transportation. Our results suggest that accessing distant labor markets through credit for investment in individual mobility yields high and persistent returns.
(2) Information Choice in Auctions

Nina Bobkova
The choice of an auction mechanism influences which object characteristics bidders learn about and whether the object is allocated efficiently. Some object characteristics are valued equally by all bidders and thus are inconsequential for the efficient allocation. Others matter only to certain bidders and thus determine the bidder with the highest valuation. I show when the second-price auction is ex ante efficient by inducing bidders to seek socially relevant information. When facing a continuous learning trade-off, bidders learn more about socially relevant components and less about common characteristics of the object in a second-price auction than a first-price auction.
Full-Text Access | Supplementary Materials  
(3) Revealing Stereotypes: Evidence from Immigrants in Schools
Alberto Alesina, Michela Carlana, Eliana La Ferrara, and Paolo Pinotti
We study how people change their behavior after being made aware of bias. Teachers in Italian schools give lower grades to immigrant students relative to natives of comparable ability. In two experiments, we reveal to teachers their own stereotypes, measured by an Implicit Association Test (IAT). In the first, we find that learning one's IAT before assigning grades reduces the native-immigrant grade gap. In the second, IAT disclosure and generic debiasing have similar average effects, but there is heterogeneity: teachers with stronger negative stereotypes do not respond to generic debiasing but change their behavior when informed about their own IAT.
Full-Text Access | Supplementary Materials  
(4) Misallocation under Trade Liberalization
Yan Bai, Keyu Jin, and Dan Lu
This paper formalizes a classic idea that in second-best environments trade can induce welfare losses: incremental income losses from distortions can outweigh trade gains. In a Melitz model with distortionary taxes, we derive sufficient statistics for welfare gains/losses and show departures from the efficient case (Arkolakis, Costinot, and Rodríguez-Clare 2012) can be captured by the gap between an input and output share and domestic extensive margin elasticities. The loss reflects an endogenous selection of more subsidized firms into exporting. Using Chinese manufacturing data in 2005 and model-inferred firm-level distortions, we demonstrate that a sizable negative fiscal externality can potentially offset conventional gains.
Full-Text Access | Supplementary Materials  
(5) Demographic Origins of the Start-up Deficit
Fatih Karahan, Benjamin Pugsley, and Ayşegül Şahin
We propose a simple explanation for the long-run decline in the US start-up rate. It originates from a slowdown in labor supply growth since the late 1970s, largely predetermined by demographics. This channel can explain roughly half of the decline and why incumbent firm survival and average growth over the life cycle have changed little. We show these results in a standard model of firm dynamics and test the mechanism using cross-state variation in labor supply growth. Finally, we show that a longer entry rate series imputed using historical establishment tabulations rises over the 1960s–1970s period of accelerating labor force growth.
Full-Text Access | Supplementary Materials  
(6) Opportunity Unraveled: Private Information and the Missing Markets for Financing Human Capital
Daniel Herbst and Nathaniel Hendren
We examine whether adverse selection has unraveled private markets for equity and state-contingent debt contracts for financing higher education. Using survey data on beliefs, we show a typical college-goer would have to repay $1.64 in present value for every $1 of financing to overcome adverse selection in an equity market. We find that risk-averse college-goers are not willing to accept these terms, so markets unravel. We discuss why moral hazard, biased beliefs, and outside credit options are less likely to explain the absence of these markets. We quantify the welfare gains for subsidizing equity-like contracts that mitigate college-going risks.
Full-Text Access | Supplementary Materials  
(7) Bank Runs, Fragility, and Credit Easing
Manuel Amador and Javier Bianchi
We present a tractable dynamic general equilibrium model of self-fulfilling bank runs, where banks trade capital in competitive and liquid markets but remain vulnerable to runs due to a loss of creditor confidence. We characterize how the vulnerability of an individual bank depends on its leverage position and the economy-wide asset prices. We study the effect of credit easing policies, in the form of asset purchases. When a banking crisis is generated by runs, credit easing can reduce the number of defaulting banks and enhance welfare. When the crisis is driven by fundamentals, credit easing may have adverse consequences.
Full-Text Access | Supplementary Materials  
(8) 100 Years of Rising Corporate Concentration
Spencer Y. Kwon, Yueran Ma, and Kaspar Zimmermann
We collect data on the size distribution of US businesses for 100 years, and use these data to estimate the concentration of production (e.g., asset share or sales share of top businesses). The data show that concentration has increased persistently over the past century. Rising concentration was stronger in manufacturing and mining before the 1970s, and stronger in services, retail, and wholesale after the 1970s. The results are robust to different measurement methods and consistent across different historical sources. Our findings suggest that large firms have become more important in the US economy for a long period of time.
Full-Text Access | Supplementary Materials  
(9) Personalized Pricing and Competition
Andrew Rhodes and Jidong Zhou
We study personalized pricing in a general oligopoly model. The impact of personalized pricing relative to uniform pricing hinges on the degree of market coverage. If market conditions are such that coverage is high (e.g., the production cost is low or the number of firms is high), personalized pricing harms firms and benefits consumers, whereas the opposite is true if coverage is low. When only some firms have data to personalize prices, consumers can be worse off compared to when either all or no firms personalize prices.
Full-Text Access | Supplementary Materials  
(10) Organized Crime and Economic Growth: Evidence from Municipalities Infiltrated by the Mafia
Alessandra Fenizia and Raffaele Saggio
This paper studies the long-run economic impact of dismissing city councils infiltrated by organized crime. Applying a matched difference-in-differences design to the universe of Italian social security records, we find that city council dismissals (CCDs) increase employment, the number of firms, and industrial real estate prices. The effects are concentrated in Mafia-dominated sectors and in municipalities where fewer incumbents are reelected. The dismissals generate large economic returns by weakening the Mafia and fostering trust in local institutions. The analysis suggests that CCDs represent an effective intervention for establishing legitimacy and spurring economic activity in areas dominated by organized crime.
Full-Text Access | Supplementary Materials  
(11) The Gift of a Lifetime: The Hospital, Modern Medicine, and Mortality
Alex Hollingsworth, Krzysztof Karbownik, Melissa A. Thomasson, and Anthony Wray
We explore how access to modern hospitals and medicine affects mortality by leveraging efforts of the Duke Endowment to modernize hospitals in the early twentieth century. The Endowment helped communities build and expand hospitals, obtain state-of-the-art medical technology, attract qualified medical personnel, and refine management practices. We find that Duke support increased the size and quality of the medical sector, fostering growth in not-for-profit hospitals and high-quality physicians. Duke funding reduced both infant mortality—with larger effects for Black infants than White infants—and long-run mortality. Finally, we find that communities aided by Duke benefited more from medical innovations.
(2) Monotone Function Intervals: Theory and Applications

Kai Hao Yang and Alexander K. Zentefis
A monotone function interval is the set of monotone functions that lie pointwise between two fixed monotone functions. We characterize the set of extreme points of monotone function intervals and apply this to a number of economic settings. First, we leverage the main result to characterize the set of distributions of posterior quantiles that can be induced by a signal, with applications to political economy, Bayesian persuasion, and the psychology of judgment. Second, we combine our characterization with properties of convex optimization problems to unify and generalize seminal results in the literature on security design under adverse selection and moral hazard.
Full-Text Access | Supplementary Materials  
(3) Long-Run Trends in Long-Maturity Real Rates, 1311–2022
Kenneth S. Rogoff, Barbara Rossi, and Paul Schmelzing
Taking advantage of key recent advances in long-run economic and financial data, we analyze the statistical properties of global long-maturity real interest rates over the past seven centuries. In contrast to existing consensus, we find that real interest rates are in fact trend stationary and exhibit a persistent downward trend since the Renaissance. We investigate structural breaks in real interest rates over time and find that overall the Black Death and the 1557 "Trinity default" appear as consistent inflection points. We further show that demographic and productivity factors do not represent convincing drivers of real interest rates over long spans.
Full-Text Access | Supplementary Materials  
(4) Indirect Effects of Access to Finance
Jing Cai and Adam Szeidl
We created experimental variation across markets in China in the share of firms having access to a new loan product. Access to finance had a large positive direct effect on the performance of treated firms but a similar-sized negative indirect effect on that of firms with treated competitors, leading to nondetectable gains in producer surplus. Access to finance had a positive direct effect on business quality and consumer satisfaction and a negative effect on price, which were not offset by indirect effects, implying net gains in consumer surplus. We document other indirect effects and combine effects in a welfare evaluation.
Full-Text Access | Supplementary Materials  
(5) Strengthening State Capacity: Civil Service Reform and Public Sector Performance during the Gilded Age
Abhay Aneja and Guo Xu
We use newly digitized records from the post office to study the effects of strengthened state capacity between 1875 and 1901. Exploiting the implementation of the Pendleton Act—a landmark statute that shielded bureaucrats from political interference—across US cities over two waves, we find that civil service reform reduced postal delivery errors and increased productivity. These improvements were most pronounced during election years when the reform dampened bureaucratic turnover. We provide suggestive evidence that reformed cities witnessed declining local partisan newspapers. Separating politics from administration, therefore, not only improved state effectiveness but also weakened the role of local politics.
Full-Text Access | Supplementary Materials  
(6) Repeated Trading: Transparency and Market Structure
Ayça Kaya and Santanu Roy
We analyze the effect of transparency of past trading volumes in markets where an informed long-lived seller can repeatedly trade with short-lived uninformed buyers. Transparency allows buyers to observe previously sold quantities. In markets with intraperiod monopsony (single buyer each period), transparency reduces welfare if the ex ante expected quality is low but improves welfare if the expected quality is high. The effect is reversed in markets with intraperiod competition (multiple buyers each period). This discrepancy in the efficiency implications of transparency is explained by how buyer competition affects the seller's ability to capture rents, which, in turn, influences market screening.
Full-Text Access | Supplementary Materials  
(7) Learning by Necessity: Government Demand, Capacity Constraints, and Productivity Growth
Ethan Ilzetzki
This paper studies how firms adapt to demand shocks when facing capacity constraints. I show that increases in government purchases raise total factor productivity in quantity units at the production line level. Productivity gains are concentrated in plants facing tighter capacity constraints, a phenomenon I call "learning by necessity." Evidence is based on newly digitized archival data on US World War II aircraft production. Shifts in demand across aircraft with different strategic roles provide an instrument for aircraft demand. I show that plants adapted to surging demand by improving production methods, outsourcing, and combating absenteeism, primarily when facing tighter capacity constraints.
Full-Text Access | Supplementary Materials  
(8) A Discrimination Report Card
Patrick Kline, Evan K. Rose, and Christopher R. Walters
We develop an empirical Bayes ranking procedure that assigns ordinal grades to noisy measurements, balancing the information content of the assigned grades against the expected frequency of ranking errors. Applying the method to a massive correspondence experiment, we grade the race and gender contact gaps of 97 US employers, the identities of which we disclose for the first time. The grades are presented alongside measures of uncertainty about each firm's contact gap in an accessible report card that is easily adaptable to other settings where ranks and levels are of simultaneous interest.
Full-Text Access | Supplementary Materials  
(9) Crowding in School Choice
William Phan, Ryan Tierney, and Yu Zhou
We consider the market design problem of matching students to schools in the presence of crowding effects. These effects are salient in parents' decision-making and the empirical literature; however, they cause difficulties in the design of satisfactory mechanisms and, as such, are not currently considered. We propose a new framework and an equilibrium notion that accommodates crowding, no-envy, and respect for priorities. The equilibrium has a student-optimal element that induces an incentive-compatible mechanism and is implementable via a novel algorithm. Moreover, analogs of fundamental structural results of the matching literature (the rural hospitals theorem, welfare lattice, etc.) survive.
Full-Text Access | Supplementary Materials  
(10) Data, Competition, and Digital Platforms
Dirk Bergemann and Alessandro Bonatti
A monopolist platform uses data to match heterogeneous consumers with multiproduct sellers. The consumers can purchase the products on the platform or search off the platform. The platform sells targeted ads to sellers that recommend their products to consumers and reveals information to consumers about their match values. The revenue-optimal mechanism is a managed advertising campaign that matches products and preferences efficiently. In equilibrium, sellers offer higher qualities at lower unit prices on than off platform. The platform exploits its information advantage to increase its bargaining power vis-à-vis the sellers. Finally, privacy-respecting data-governance rules can lead to welfare gains for consumers.
Full-Text Access | Supplementary Materials  
(11) Buying from a Group
Nima Haghpanah, Aditya Kuvalekar, and Elliot Lipnowski
A buyer procures a good owned by a group of sellers whose heterogeneous cost of trade is private information. The buyer must either buy the whole good or nothing, and sellers share the transfer in proportion to their share of the good. We characterize the optimal mechanism: trade occurs if and only if the buyer's benefit of trade exceeds a weighted average of sellers' virtual costs. These weights are endogenous, with sellers who are ex ante less inclined to trade receiving higher weight. This mechanism always outperforms posted-price mechanisms. An extension characterizes the entire Pareto frontier.
(2) Arbitraging Covered Interest Rate Parity Deviations and Bank Lending

Lorena Keller
I propose and test a new channel through which covered interest rate parity (CIP) deviations can affect bank lending in emerging economies. I argue that when CIP deviations exist, banks attempt to arbitrage them. To do so, banks must borrow in a particular currency. When this currency is scarce, bank lending in the currency required to arbitrage decreases, while they use this currency in their arbitrage activities. I test this channel by exploiting differences in the abilities of Peruvian banks to arbitrage CIP deviations. I find evidence that supports the proposed channel.
Full-Text Access | Supplementary Materials  
(3) Revealing Choice Bracketing
Andrew Ellis and David J. Freeman
Experiments suggest that people fail to take into account interdependencies between their choices—they do not broadly bracket. Researchers often instead assume people narrowly bracket, but existing designs do not test it. We design a novel experiment and revealed preference tests for how someone brackets their choices. In portfolio allocation under risk, social allocation, and induced-value shopping experiments, 40–43 percent of subjects are consistent with narrow bracketing, and 0–16 percent with broad bracketing. Adjusting for each model's predictive precision, 74 percent of subjects are best described by narrow bracketing, 13 percent by broad bracketing, and 6 percent by intermediate cases.
Full-Text Access | Supplementary Materials  
(4) Valuing Long-Term Property Rights with Anticipated Political Regime Shifts
Zhiguo He, Maggie Hu, Zhenping Wang, and Vincent Yao
We identify exposure to political risk by exploiting a unique variation around land lease extension protection after 2047 in Hong Kong's housing market due to historical arrangements. Relative to properties that have been promised an extension protection, those with unprotected leases granted by the current government are sold at a discount of 8 percent; those with colonial leases suffer an additional discount of 8 percent. Incorporating estimated structural parameters that suggest an additional 20 percent ground rent after 2047, our model matches empirical discounts well across lease horizons. Discounts increase over time, particularly in areas with greater pessimism about the city's future.
Full-Text Access | Supplementary Materials  
(5) Age Set versus Kin: Culture and Financial Ties in East Africa
Jacob Moscona and Awa Ambra Seck
We study how social organization shapes patterns of economic interaction and the effects of national policy, focusing on the distinction between age-based and kin-based groups in sub-Saharan Africa. Motivated by ethnographic accounts suggesting that this distinction affects redistribution, we analyze a cash transfer program in Kenya and find that in age-based societies there are consumption spillovers within the age cohort, but not the extended family, while in kin-based societies we find the opposite. Next, we document that social structure shapes the impact of policy by showing that Uganda's pension program had positive effects on child nutrition only in kin-based societies.
Full-Text Access | Supplementary Materials  
(6) Investing in the Next Generation: The Long-Run Impacts of a Liquidity Shock
Patrick Agte, Arielle Bernhardt, Erica Field, Rohini Pande, and Natalia Rigol
Poor entrepreneurs must frequently choose between business investment and children's education. To examine this trade-off, we exploit experimental variation in short-run microenterprise growth among a sample of Indian households and track schooling and business out-comes over eleven years. Treated households, who experience higher initial microenterprise growth, are on average one-third more likely to send children to college. However, educational investment and schooling gains are concentrated among literate-parent households, whose enterprises eventually stagnate. In contrast, illiterate-parent households experience long-run business gains but declines in children's education. Our findings suggest that microenterprise growth has the potential to reduce relative intergenerational educational mobility.
Full-Text Access | Supplementary Materials  
(7) Big Loans to Small Businesses: Predicting Winners and Losers in an Entrepreneurial Lending Experiment
Gharad Bryan, Dean Karlan, and Adam Osman
We experimentally study the impact of relatively large enterprise loans in Egypt. Larger loans generate small average impacts, but machine learning using psychometric data reveals "top performers" (those with the highest predicted treatment effects) substantially increase profits, while profits drop for poor performers. The large differences imply that lender credit allocation decisions matter for aggregate income, yet we find existing practice leads to substantial misallocation. We argue that some entrepreneurs are overoptimistic and squander the opportunities presented by larger loans by taking on too much risk, and show the promise of allocations based on entrepreneurial type relative to firm characteristics.
Full-Text Access | Supplementary Materials  
(8) In-Kind Transfers as Insurance
Lucie Gadenne, Samuel Norris, Monica Singhal, and Sandip Sukhtankar
Households in developing countries often face variation in the prices of consumption goods. We develop a model demonstrating that in-kind transfers will provide insurance benefits against price risk if the covariance between the marginal utility of income and price is positive. Using calorie shortfalls as a proxy for marginal utility, we find that this condition holds for low-income Indian households. Expansions in India's flagship in-kind food transfer program not only increase caloric intake but also reduce caloric sensitivity to prices. Our results contribute to ongoing debates about the optimal form of social protection programs.
Full-Text Access | Supplementary Materials  
(9) Spending and Job-Finding Impacts of Expanded Unemployment Benefits: Evidence from Administrative Micro Data
Peter Ganong, Fiona Greig, Pascal Noel, Daniel M. Sullivan, and Joseph Vavra
We show that the largest increase in unemployment benefits in US history had large spending impacts and small job-finding impacts. This finding has three implications. First, increased benefits were important for explaining aggregate spending dynamics—but not employment dynamics—during the pandemic. Second, benefit expansions allow us to study the MPC of normally low-liquidity households in a high-liquidity state. These households still have high MPCs. This suggests a role for permanent behavioral characteristics, rather than just current liquidity, in driving spending behavior. Third, the mechanisms driving our results imply that temporary benefit supplements are a promising countercyclical tool.
Full-Text Access | Supplementary Materials  
(10) Experimentation in Networks
Simon Board and Moritz Meyer-ter-Vehn
We propose a model of strategic experimentation on social networks in which forward-looking agents learn from their own and neighbors' successes. In equilibrium, private discovery is followed by social diffusion. Social learning crowds out own experimentation, so total information decreases with network density; we determine density thresholds below which agents' asymptotic learning is perfect. By contrast, agent welfare is single peaked in network density and achieves a second-best benchmark level at intermediate levels that strike a balance between discovery and diffusion.
Full-Text Access | Supplementary Materials  
(11) Comparisons of Signals
Benjamin Brooks, Alexander Frankel, and Emir Kamenica
A signal is a description of an information source that specifies both its correlation with the state and its correlation with other signals. Extending Blackwell (1953), we characterize when one signal is more valuable than another regardless of preferences and regardless of access to other signals. This comparison is equivalent to reveal-or-refine: every realization of the more valuable signal reveals the state or refines the realization of the less valuable signal. We also study other comparisons of signals, including sufficiency, martingale dominance, and Lehmann. Reveal-or-refine is also equivalent to making any of these comparisons robust to access to other signals.

*群友可直接前往社群下载AER原文PDF。

下面这些短链接文章属于合集,可以收藏起来阅读,不然以后都找不到了。

7年,计量经济圈近2000篇不重类计量文章,

可直接在公众号菜单栏搜索任何计量相关问题,

Econometrics Circle




数据系列空间矩阵 | 工企数据 | PM2.5 | 市场化指数 | CO2数据 |  夜间灯光 官员方言  | 微观数据 | 内部数据
计量系列匹配方法 | 内生性 | 工具变量 | DID | 面板数据 | 常用TOOL | 中介调节 | 时间序列 | RDD断点 | 合成控制 | 200篇合辑 | 因果识别 | 社会网络 | 空间DID
数据处理Stata | R | Python | 缺失值 | CHIP/ CHNS/CHARLS/CFPS/CGSS等 |
干货系列能源环境 | 效率研究 | 空间计量 | 国际经贸 | 计量软件 | 商科研究 | 机器学习 | SSCI | CSSCI | SSCI查询 | 名家经验
计量经济圈组织了一个计量社群,有如下特征:热情互助最多前沿趋势最多、社科资料最多、社科数据最多、科研牛人最多、海外名校最多。因此,建议积极进取和有强烈研习激情的中青年学者到社群交流探讨,始终坚信优秀是通过感染优秀而互相成就彼此的。